You are on page 1of 68

OPOSA VS. FACTORAN, JR.

224 SCRA 792


Facts:
The principal petitioners are all minors duly represented and joined by their
respective parents. Impleaded as an additional plaintiff is the Philippine Ecological
Network, Inc. (PENI), a domestic, non-stock and non-profit corporation organized for the
purpose of, inter alia, engaging in concerted action geared for the protection of our
environment and natural resources. The original defendant was the Honorable
Fulgencio S. Factoran, Jr., then Secretary of the Department of Environment and
Natural Resources (DENR). His substitution in this petition by the new Secretary, the
Honorable Angel C. Alcala, was subsequently ordered upon proper motion by the
petitioners.
The complaint was instituted as a taxpayers' class suit and alleges that the
plaintiffs "are all citizens of the Republic of the Philippines, taxpayers, and entitled to the
full benefit, use and enjoyment of the natural resource treasure that is the country's
virgin tropical forests." The same was filed for themselves and others who are equally
concerned about the preservation of said resource but are "so numerous that it is
impracticable to bring them all before the Court."
The minors further asserts that they "represent their generation as well as
generations yet unborn."
The complaint starts off with the general averments that the Philippine
archipelago of 7,100 islands has a land area of thirty million (30,000,000) hectares and
is endowed with rich, lush and verdant rainforests in which varied, rare and unique
species of flora and fauna may be found. These rainforests contain a genetic, biological
and chemical pool which is irreplaceable; they are also the habitat of indigenous
Philippine cultures which have existed, endured and flourished since time immemorial.
Scientific evidence reveals that in order to maintain a balanced and healthful ecology,
the country's land area should be utilized on the basis of a ratio of fifty-four per cent
(54%) for forest cover and forty-six per cent (46%) for agricultural, residential, industrial,
commercial and other uses. The distortion and disturbance of this balance as a
consequence of deforestation have resulted in a host of environmental tragedies.
Plaintiffs further assert that the adverse and detrimental consequences of
continued and deforestation are so capable of unquestionable demonstration that the
same may be submitted as a matter of judicial notice.
On 22 June 1990, the original defendant, Secretary Factoran, Jr., filed a Motion
to Dismiss the complaint based on two (2) grounds, namely: (1) the plaintiffs have no
cause of action against him and (2) the issue raised by the plaintiffs is a political

question which properly pertains to the legislative or executive branches of


Government.
In their 12 July 1990 Opposition to the Motion, the petitioners maintain that (1)
the complaint shows a clear and unmistakable cause of action, (2) the motion is dilatory
and (3) the action presents a justiciable question as it involves the defendant's abuse of
discretion.
On 18 July 1991, respondent Judge issued an order granting the aforementioned
motion to dismiss. In the said order, not only was the defendant's claim that the
complaint states no cause of action against him and that it raises a political question
sustained, the respondent Judge further ruled that the granting of the relief prayed for
would result in the impairment of contracts which is prohibited by the fundamental law of
the land.
Plaintiffs thus filed the instant special civil action for certiorari under Rule 65 of
the Revised Rules of Court and asked the Court to rescind and set aside the dismissal
order on the ground that the respondent Judge gravely abused his discretion in
dismissing the action.
Petitioners contend that the complaint clearly and unmistakably states a cause of
action as it contains sufficient allegations concerning their right to a sound environment,
the right of the people to a balanced and healthful ecology, the concept of generational
genocide and the concept of man's inalienable right to self-preservation and selfperpetuation. Rely on the respondent's correlative obligation per Section 4 of E.O. No.
192, to safeguard the people's right to a healthful environment. It is further claimed that
the issue of the respondent Secretary's alleged grave abuse of discretion in granting
Timber License Agreements (TLAs) to cover more areas for logging than what is
available involves a judicial question. Non-impairment clause does not apply in this case
because TLAs are not contracts, even if TLAs may be considered protected by the said
clause, it is well settled that they may still be revoked by the State when the public
interest so requires.
Respondents avers that the petitioners failed to allege in their complaint a
specific legal right violated by the respondent Secretary for which any relief is provided
by law. They see nothing in the complaint but vague and nebulous allegations
concerning an "environmental right" which supposedly entitles the petitioners to the
"protection by the state in its capacity as parens patriae." Such allegations, according to
them, do not reveal a valid cause of action. They then reiterate the theory that the
question of whether logging should be permitted in the country is a political question
which should be properly addressed to the executive or legislative branches of
Government. They therefore assert that the petitioners' resources is not to file an action
to court, but to lobby before Congress for the passage of a bill that would ban logging
totally.

As to the matter of the cancellation of the TLAs, respondents submit that the
same cannot be done by the State without due process of law. Once issued, a TLA
remains effective for a certain period of time usually for twenty-five (25) years. During its
effectivity, the same can neither be revised nor cancelled unless the holder has been
found, after due notice and hearing, to have violated the terms of the agreement or
other forestry laws and regulations. Petitioners' proposition to have all the TLAs
indiscriminately cancelled without the requisite hearing would be violative of the
requirements of due process.
It is prayed for that judgment be rendered ordering defendant, his agents,
representatives and other persons acting in his behalf to:
(1) Cancel all existing timber license agreements (TLAs) in the country;
(2) Cease and desist from receiving, accepting, processing, renewing or approving new
TLAs, and
(3) Granting the plaintiffs such other reliefs just and equitable under the premises.

Issue:
Whether or not the petitioners have a cause of action to prevent the
misappropriation or impairment of Philippine rainforests and arrest the unabated
hemorrhage of the country's vital life support systems and continued rape of Mother
Earth.
Ruling:
The instant Petition is granted, and the challenged Order of respondent Judge is
set aside. The petitioners may therefore amend their complaint to implead as
defendants the holders or grantees of the questioned timber license agreements.
LOCUS STANDI:
The said civil case is indeed a class suit. The subject matter of the complaint is of
common and general interest not just to several, but to all citizens of the Philippines.
Consequently, since the parties are so numerous, it, becomes impracticable, if
not totally impossible, to bring all of them before the court. The SC likewise declares
that the plaintiffs therein are numerous and representative enough to ensure the full
protection of all concerned interests. Hence, all the requisites for the filing of a valid
class suit under Section 12, Rule 3 of the Revised Rules of Court are present both in
the said civil case and in the instant petition, the latter being but an incident to the
former.

This case, however, has a special and novel element. Petitioners minors assert
that they represent their generation as well as generations yet unborn. The SC finds no
difficulty in ruling that they can, for themselves, for others of their generation and for the
succeeding generations, file a class suit. Their personality to sue in behalf of the
succeeding generations can only be based on the concept of intergenerational
responsibility insofar as the right to a balanced and healthful ecology is concerned.
Such a right considers the "rhythm and harmony of nature."
Nature means the created world in its entirety.Such rhythm and harmony
indispensably include, inter alia, the judicious disposition, utilization, management,
renewal and conservation of the country's forest, mineral, land, waters, fisheries,
wildlife, off-shore areas and other natural resources to the end that their exploration,
development and utilization be equitably accessible to the present as well as future
generations. Needless to say, every generation has a responsibility to the next to
preserve that rhythm and harmony for the full enjoyment of a balanced and healthful
ecology.
The minors' assertion of their right to a sound environment constitutes, at the
same time, the performance of their obligation to ensure the protection of that right for
the generations to come.
RIGHT TO A BALANCED AND HEALTHFUL ECOLOGY:
The complaint focuses on the right to a balanced and healthful ecology which, for
the first time in our nation's constitutional history, is solemnly incorporated in the
fundamental law (Section 16, Article II of the 1987 Constitution).
This right unites with the right to health which is provided for in the Section 15 of
the same article.
While the right to a balanced and healthful ecology is to be found under the
Declaration of Principles and State Policies and not under the Bill of Rights, it does not
follow that it is less important than any of the civil and political rights enumerated in the
latter. Such a right belongs to a different category of rights altogether for it concerns
nothing less than self-preservation and self-perpetuation the advancement of which may
even be said to predate all governments and constitutions. As a matter of fact, these
basic rights need not even be written in the Constitution for they are assumed to exist
from the inception of humankind.
If they are now explicitly mentioned in the fundamental charter, it is because of
the well-founded fear of its framers that unless the rights to a balanced and healthful
ecology and to health are mandated as state policies by the Constitution itself, thereby
highlighting their continuing importance and imposing upon the state a solemn
obligation to preserve the first and protect and advance the second, the day would not
be too far when all else would be lost not only for the present generation, but also for
those to come generations which stand to inherit nothing but parched earth incapable of
sustaining life.
4

The right to a balanced and healthful ecology carries with it the correlative duty to
refrain from impairing the environment. The said right implies, among many other
things, the judicious management and conservation of the country's forests. Without
such forests, the ecological or environmental balance would be irreversibly disrupted.
Conformably with the enunciated right to a balanced and healthful ecology and
the right to health, then President Corazon C. Aquino promulgated on 10 June 1987
E.O. No. 192, Section 4 of which expressly mandates that the Department of
Environment and Natural Resources "shall be the primary government agency
responsible for the conservation, management, development and proper use of the
country's environment and natural resources, specifically forest and grazing lands,
mineral, resources, including those in reservation and watershed areas, and lands of
the public domain, as well as the licensing and regulation of all natural resources as
may be provided for by law in order to ensure equitable sharing of the benefits derived
there from for the welfare of the present and future generations of Filipinos."
This policy declaration is substantially re-stated it Title XIV, Book IV of the
Administrative Code of 1987. It stresses "the necessity of maintaining a sound
ecological balance and protecting and enhancing the quality of the environment."
Section 2 of the same Title, on the other hand, specifically speaks of the mandate of the
DENR; however, it makes particular reference to the fact of the agency's being subject
to law and higher authority.
Both E.O. NO. 192 and the Administrative Code of 1987 have set the objectives
which will serve as the bases for policy formulation, and have defined the powers and
functions of the DENR.
On 6 June 1977, P.D. No. 1151 (Philippine Environmental Policy) and P.D. No.
1152 (Philippine Environment Code) were issued. As its goal, it speaks of the
"responsibilities of each generation as trustee and guardian of the environment for
succeeding generations." The latter statute, on the other hand, gave flesh to the said
policy.
Thus, the right of the petitioners (and all those they represent) to a balanced and
healthful ecology is as clear as the DENR's duty under its mandate and by virtue of
its powers and functions under E.O. No. 192 and the Administrative Code of 1987 to
protect and advance the said right.
A denial or violation of that right by the other who has the correlative duty or obligation
to respect or protect the same gives rise to a cause of action.
CAUSE OF ACTION:
A cause of action is defined as:

. . . an act or omission of one party in violation of the legal right or rights of the other;
and its essential elements are legal right of the plaintiff, correlative obligation of the
defendant, and act or omission of the defendant in violation of said legal right.
The question submitted to the court for resolution involves the sufficiency of the facts
alleged in the complaint itself. Falsity of the said allegations is beside the point for the
truth thereof is deemed hypothetically admitted. It bears stressing, however, that insofar
as the cancellation of the TLAs is concerned, there is the need to implead, as party
defendants, the grantees thereof for they are indispensable parties.
NON-IMPAIRMENT OF CONTRACTS:
The last ground invoked by the trial court in dismissing the complaint is the nonimpairment of contracts clause found in the Constitution.
The court declared that to cancel all existing timber license agreements in the
country and to cease and desist from receiving, accepting, processing, renewing or
approving new timber license agreements amount to impairment of contracts abhorred
by the fundamental law.
The respondent Secretary did not even invoke in his motion to dismiss the nonimpairment clause. If he had done so, he would have acted with utmost infidelity to the
Government by providing undue and unwarranted benefits and advantages to the
timber license holders because he would have forever bound the Government to strictly
respect the said licenses according to their terms and conditions regardless of changes
in policy and the demands of public interest and welfare.
Section 20 of the Forestry Reform Code (P.D. No. 705) which provides:
. . . Provided, That when the national interest so requires, the
President may amend, modify, replace or rescind any contract,
concession, permit, licenses or any other form of privilege granted
herein . . .
Tan vs. Director of Forestry:. . .A timber license is an instrument by
which the State regulates the utilization and disposition of forest
resources to the end that public welfare is promoted. A timber
license is not a contract within the purview of the due process
clause; it is only a license or privilege, which can be validly
withdrawn whenever dictated by public interest or public welfare as
in this case.
Since timber licenses are not contracts, the non-impairment clause cannot be invoked.
Even if it is to be assumed that the same are contracts, the instant case does not
involve a law or even an executive issuance declaring the cancellation or modification of
existing timber licenses. Hence, the non-impairment clause cannot as yet be invoked.

Abe vs. Foster Wheeler Corp.:The freedom of contract, under our


system of government, is not meant to be absolute. The same is
understood to be subject to reasonable legislative regulation
aimed at the promotion of public health, moral, safety and
welfare. In other words, the constitutional guaranty of nonimpairment of obligations of contract is limited by the exercise of
the police power of the State, in the interest of public health,
safety, moral and general welfare.
In short, the non-impairment clause must yield to the police power of the state.

MERIDA VS. PEOPLE OF THE PHILIPPINES


554 SCRA 366
Facts:
Petitioner was charged in the RTC of Romblon with violation of Section 68 of PD
705 for "cutting, gathering, collecting and removing a lone narra tree inside a private
land over which private complainant Oscar Tansiongco claims ownership. When
confronted during the meeting about the felled narra tree, petitioner admitted cutting the
tree but claimed that he did so with the permission of one Vicar Calix who, according to
petitioner, bought the Mayod Property from Tansiongco in October 1987 under a pacto
de retro sale. It was later found out that he converted the narra trunk into lumber.
He was found guilty by the Trial Court but he appealed to the Court of Appeals
reiterating his defense of denial. Petitioner also contended that the trial court did not
acquire jurisdiction over the case because it was based on a complaint filed by
Tansiongco and not by a forest officer as provided under Section 80 of PD 705. CA
affirmed the lower courts ruling, but ordered the seized lumber confiscated in the
government's favor. Also, it sustained the trial court's finding that petitioner is bound by
his extrajudicial admissions of cutting the narra tree in the Mayod Property without any
DENR permit.
Issues:
Whether or not the trial court acquired jurisdiction over Criminal Case No. 2207
even though it was based on a complaint filed by Tansiongco and not by a DENR
forest officer.
Whether or not petitioner is liable for violation of Section 68 of PD 705.
Ruling:
1. The Revised Rules of Criminal Procedure list the cases which must be initiated
by a complaint filed by specified individuals, non-compliance of which ousts the trial
court of jurisdiction from trying such cases. However, these cases concern only
defamation and other crimes against chastity and not to cases concerning Section 68 of
PD 705. Further, Section 80 of PD 705 does not prohibit an interested person from filing
a complaint before any qualified officer for violation of Section 68 of PD 705, as
amended.
Moreover, here, it was not "forest officers or employees of the Bureau of Forest
Development who reported to Hernandez the tree-cutting in the Mayod Property but
Tansiongco, a private citizen who claims ownership over the Mayod Property. Thus,
Hernandez cannot be faulted for not conducting an investigation to determine "if there is
prima facie evidence to support the complaint or report." At any rate, Tansiongco was
not precluded, either under Section 80 of PD 705 or the Revised Rules, from filing a
complaint before the Provincial Prosecutor for petitioner's alleged violation of Section 68
of PD 705.
8

2. Petitioner is guilt of the second paragraph of section 80, which is the cutting,
gathering, collecting, or removing of timber from alienable or disposable public land, or
from private land without any authority. The court also said that the lumber or
processed log is covered by the forest products term in PD 705, as the law does not
distinguish between a raw and processed timber.

MOMONGAN VS. JUDGE OMIPON


242 SCRA 332
Facts:
At around 10:00 oclock of November 14, 1992, police officers of the Municipality
of Hinunangan, Southern Leyte apprehended Dionisio Golpe while he was driving his
truck loaded with illegally cut lumber. The truck and logs were impounded. A complaint
was filed against Basilio Cabig, the alleged owner of the logs. After conducting the
preliminary investigation, respondent Judge Rafael B. Omipon found that a prima facie
case exists against Mr. Cabig but he ordered the release of the truck inasmuch as the
owner/driver, Mr. Golpe, was not charged in the complaint.
Regional Director Augustus L. Momongan of the Department of Environment and
Natural Resources filed the instant complaint against respondent Judge alleging that his
order releasing the truck used in the transport of illegally cut forest products violated
Presidential Decree 705, as amended by Executive Order No. 277, Section 68 and 68A1 and Administrative Order No.59 series of 1990. Complainant claims that respondent
Judge has no authority to order the release of the truck despite the non-inclusion of Mr.
Golpe in the complaint. The truck should have been turned over to the Community
Environment and Natural Resources Office of San Juan, Southern Leyte for appropriate
disposition as the same falls under the administrative jurisdiction of the Department of
Environment and Natural Resources Office.
Respondent Judge explained that after conducting the preliminary investigation,
he found that Golpe, the owner of the truck, is principally engaged in the hauling of sand
and gravel and the delivery of hollow blocks. On his way home after delivering hollow
blocks in Barangay Sto. Nino II, he met his friend Cabig who requested him to load
sliced lumber and deliver the same at Brgy. Lungsod-daan, Hinundayan to be used for
the construction of a barangay high school building. They were apprehended when the
truck had a flat tire. After changing the tire, both the lumber and the truck were ordered
deposited at the police station of Hinunangan.
Respondent Judge observed that Golpe has a lesser participation in the crime of
illegal logging and, being a mere accessory, he might be utilized by the Acting Chief of
Police as prosecution witness against Cabig. More importantly, the fact that the
complaint charged only Cabig, respondent Judge, in the exercise of his sound
discretion, ordered the release of the truck owned by Golpe.
Issue:
Whether or not the order of the release of the tuck by the respondent Judge is
legally justifiable.
Ruling:
The court find respondent Judges order to release the truck owned and driven
by Mr. Dionisio Golpe legally justifiable, hence, he is not subject to any disciplinary
sanction. According to the Revised Penal Code, Art. 45, first paragraph: Every penalty
imposed for the commission of a felony shall carry with it the forfeiture of the proceeds
10

of the crime and the instrument or tools with which it was committed. However, this
cannot be done if such proceeds and instruments or tools be the property of a third
person not liable for the offense. In this case, the truck, though used to transport the
illegally cut lumber, cannot be confiscated and forfeited in the event accused therein be
convicted because the truck owner/driver, Mr. Dionisio Golpe was not indicted. Hence,
there was no justification for respondent Judge not to release the truck.
Complainant is correct in pointing out that based on Pres. Decree No. 705, Sec.
68-A and Adm. Order No. 59, the DENR Secretary or his duly authorized representative
has the power to confiscate any illegally obtained or gathered forest products and all
conveyances used in the commission of the offense and to dispose of the same in
accordance with pertinent laws. However, as complainant himself likewise pointed out,
this power is in relation to the administrative jurisdiction of the DENR.
The court do not find that when respondent Judge released the truck after he
conducted the preliminary investigation and satisfied himself that there was no reason
to continue keeping the truck, he violated Pres. Decree No. 705 and Adm. Order No. 59.
The release of the truck did not render nugatory the administrative authority of the
DENR Secretary. The confiscation proceedings under Adm. Order No. 59 is different
from the confiscation under the Revised Penal Code, which is an additional penalty
imposed in the event of conviction. Despite the order of release, the truck can be seized
again either by filing a motion for reinvestigation and motion to include the truck
owner/driver as co-accused, which complainant has done as manifested before the
lower court or by enforcing Adm. Order No. 59. Section 12 thereof categorically states
that the confiscation of the conveyance under these regulations shall be without
prejudice to any criminal action which shall be filed against the owner thereof or any
person who used the conveyance in the commission of the offense.
Under Sec. 4 of Adm. Order No. 59, if the apprehension is not made by DENR
field offices, deputized military personnel and officials of other agency apprehending
illegal logs and other forest products and their conveyances shall notify the nearest
DENR field offices and turn over said forest products and conveyances for proper action
and disposition. A period of about two weeks lapsed from the time the seizure was
made before a complaint was filed. During this period, the apprehending policemen had
enough time to turn over the logs and the truck to the nearest DENR field office for
proper action and disposition since the duty to turn over the truck to the nearest DENR
field office rests on the officials apprehending the illegal logs. There being no mandatory
duty on the part of respondent Judge to turn over the truck, he should not be visited with
disciplinary sanction when he did not refer the same to the DENR field office in San
Juan, Southern Leyte.
The Court takes this opportunity to enjoin the National Police, the DENR, the
prosecutors, and the members of the bench to coordinate with each other for a
successful campaign against illegal logging. It behooves all the concerned agencies to
seriously strive for the attainment of the constitutionally-declared policy to protect and
advance the right of the people to a balanced and healthful ecology in accord with the
rhythm and harmony of nature in order to preserve our natural resources for the benefit
of the generations still to come.
Thus, the complaint was dismissed.
11

PROVIDENT TREE FARMS V. BATARIO


231 SCRA 463
Facts:
Petitioner PTFI is a Philippine corporation engaged in industrial tree planting. It
grows gubas trees in its plantations which it supplies to a local match manufacturer
solely for production of matches. In consonance with the state policy to encourage
qualified persons to engage in industrial tree plantation, Sec. 36, par. (1), of the Revised
Forestry Code confers on entities like PTFI a set of incentives among which is a
qualified ban against importation of wood and "wood-derivated" products.
On 5 April 1989, private respondent A. J. International Corporation (AJIC)
imported 4 containers of matches from Indonesia and 2 or more containers of matches
from Singapore. On 25 April 1989, upon request of PTFI, Secretary Factoran of the
DENR issued a certification that "there are enough available softwood supply in the
Philippines for the match industry at reasonable price." In light of this, PTFI filed with
the RTC of Manila a complaint for injunction and damages with prayer for a TRO
against respondents Commissioner of Customs and AJIC to enjoin the latter from
importing matches and "wood-derivated" products, and the Collector of Customs from
allowing and releasing the importations.
The case was raffled to respondent Judge Demetrio M. Batario. Lower court
ruled in favor of respondents, stating that it had "no jurisdiction to determine what are
legal or illegal importations."
PTFI claims that what was brought before the trial court was a civil case for
injunction, "restraining the entry of safety matches into the country for the purpose of
securing compliance with Sec. 36 (l) of the Forestry Code" and for damages, "to seek
redress of its right which has been clearly violated by the importation of safety matches,
is a denial to the petitioner of the protection and incentive granted it by Section 36 (l) of
the Forestry Code. PTFI asserts the inapplicability of the procedures outlined in R.A. No.
1125 relative to incidents before the Court of Tax Appeals because the instant action is
not a protest case where the aggrieved party is not an importer. It then argues that since
it could not avail of the remedies afforded by the Tariff and Customs Code, resort to the
courts is warranted, citing Commissioner of Customs v. Alikpala.
Issue:
Whether or not the Commissioner of Customs under Sec. 1207 of the Tariff and
Customs Code and not the regular court, has "exclusive jurisdiction to determine the
legality of an importation, and other incidental matters relating to such.

12

Ruling:
Yes.Petitioner anchors his complaint on a statutory privilege or incentive granted
under Sec. 36, par. (l), of the Revised Forestry Code. The only subject of this incentive
is a ban against importation of wood, wood products or wood-derivated products which
is to be enforced by the Bureau of Customs since it has, under the Tariff and Customs
Code, the exclusive original jurisdiction over seizure and forfeiture cases and, in fact, it
is the duty of the Collector of Customs to exercise jurisdiction over prohibited
importations.
The enforcement of the importation ban under Sec. 36, par. (l), of the Revised
Forestry Code is within the exclusive realm of the Bureau of Customs, and direct
recourse of petitioner to the Regional Trial Court to compel the Commissioner of
Customs to enforce the ban is devoid of any legal basis. An order of a judge to
impound, seize or forfeit must inevitably be based on his determination and declaration
of the invalidity of the importation, hence, an usurpation of the prerogative and an
encroachment on the jurisdiction of the Bureau of Customs.
Also, PTFI's correspondence with the Bureau of Customs contesting the legality
of match importations may already take the nature of an administrative proceeding the
pendency of which would preclude the court from interfering with it under the doctrine of
primary jurisdiction.

13

PEOPLE VS. CFI QUEZON CITY, BRANCH VII


206 SCRA 187
Facts:
The private respondents were charged with the crime of qualified theft of logs,
defined and punished under Section 68 of Presidential Decree No. 705, otherwise
known as the Revised Forestry Code of the Philippines. The information provided that
Godofredo Arrozal and Luis Flores, together with 20 other John Does whose identities
are still unknown, the first-named accused being the administrator of the Infanta
Logging Corporation, conspired and entered the privately-owned land of one
Felicitacion Pujalte, titled in the name of her deceased father, Macario Prudente, and
proceeded to illegally cut, gather, and take, therefrom, without the consent of the said
owner and without any authority under a license agreement, 60 logs of different species.
On March 23, 1977, the named accused filed a motion to quash the information
on 2 grounds, to wit: (1) that the facts charged do not constitute an offense; and, (2) that
the information does not conform substantially to the prescribed form. Trial court thus
dismissed the information based on the respondents grounds.
Issue:
Whether or not the information correctly and properly charged an offense and
WON the trial court had jurisdiction over the case.
Ruling:
Yes. The elements of the crime of qualified theft of logs are: 1) That the accused
cut, gathered, collected or removed timber or other forest products; 2) that the timber or
other forest products cut, gathered, collected or removed belongs to the government or
to any private individual; and 3) that the cutting, gathering, collecting or removing was
without authority under a license agreement, lease, license, or permit granted by the
state. The failure of the information to allege that the logs taken were owned by the
state is not fatal. It should be noted that the logs subject of the complaint were taken not
from a public forest but from a private woodland registered in the name of complainant's
deceased father, Macario Prudente. The fact that only the state can grant a license
agreement, license or lease does not make the state the owner of all the logs and
timber products produced in the Philippines including those produced in private
woodlands. Thus, ownership is not an essential element of the offense as defined in
Section 60 of P.D. No. 705.
As to the second issue raised, the regular courts still has jurisdiction. Sec. 80 of
PD 705 covers 2 specific instances when a forest officer may commence a prosecution
for the violation of the Revised Forestry Code of the Philippines. The first authorizes a
forest officer or employee of the Bureau of Forestry to arrest without a warrant, any
person who has committed or is committing, in his presence, any of the offenses
described in the decree. The second covers a situation when an offense described in
14

the decree is not committed in the presence of the forest officer or employee and the
commission is brought to his attention by a report or a complaint.
In both cases, however, the forest officer or employee shall investigate the offender and
file a complaint with the appropriate official authorized by law to conduct a preliminary
investigation and file the necessary informations in court. Unfortunately, the instant case
do not fall under any of the situations covered by Section 80 of P.D. 705.
The alleged offense was committed not in the presence of a forest officer and
neither was the alleged commission reported to any forest officer. The offense was
committed in a private land and the complaint was brought by a private offended party
to the fiscal. As such, the OSG was correct in insisting that P.D. 705 did not repeal
Section 1687 of the Administrative Code giving authority to the fiscal to conduct
investigation into the matter of any crime or misdemeanor and have the necessary
information or complaint prepared or made against persons charged with the
commission of the crime.
In short, Sec. 80 does not grant exclusive authority to the forest officers, but only
special authority to reinforce the exercise of such by those upon whom it is vested by
general law.

15

AQUINO VS. PEOPLE OF THE PHILIPPINES


594 SCRA 332
Facts:
On behalf of Teachers Camp, Sergio Guzman filed with the Department of
Environment and Natural Resources an application to cut down 14 dead Benguet pine
trees within the Teachers Camp in Baguio City. The trees, which had a total volume of
13.37 cubic meters, were to be used for the repairs of Teachers Camp.
On May 19, 1993, before the issuance of the permit, a forest ranger of the Forest
Section of the Office of the City Architect and Parks Superintendent of Baguio City,
conducted an inspection of the trees to be cut. Thereafter, Sabado T. Batcagan,
Executive Director of the DENR, issued a permit allowing the cutting of 14 trees subject
to certain conditions.
On July 23, 1993, Forest Rangers received information that pine trees were
being cut at Teachers Camp without proper authority. They proceeded to the site where
they found Ernesto Aquino (petitioner), a forest ranger from CENRO, and Cuteng
supervising the cutting of the trees. They also found sawyers Benedicto Santiago and
Mike Masing on the site, together with Salinas and Nacatab who were also supervising
the cutting of the trees. The forest rangers found 23 tree stumps, out of which only 12
were covered by the permit. The volume of the trees cut with permit was 13.58 cubic
meters while the volume of the trees cut without permit was 16.55 cubic meters.
An Information for violation of Section 68 of Presidential Decree No. 705 (PD
705) was filed against petitioner, Cuteng, Nacatab, Masing, and Santiago.
Masing alleged that he was not aware of the limitations on the permit as he was
not given a copy of the permit. Masing stated that he cut 10 pine trees under the
supervision of petitioner who claimed to be in possession of the necessary
permit. Santiago also testified that he cut trees under petitioners supervision. He stated
that petitioner was in possession of the permit. While also Salinas testified that Masing
and Santiago were merely hired as sawyers and they merely followed petitioners
instructions.
Cuteng testified that he was part of the team that inspected the trees to be cut
before the permit was issued. He stated that the trees cut by Santiago were covered by
the permit.
Petitioner alleged that he was sent to supervise the cutting of trees at Teachers
Camp. He allegedly informed his superior, Paul Apilis, that he was not aware of the
trees covered by the permit. However, he still supervised the cutting of trees without
procuring a copy of the vicinity map used in the inspection of the trees to be cut. He
claimed that he could not prevent the overcutting of trees because he was just alone
while Cuteng and Santiago were accompanied by three other men.
Issue:
Whether or not the Petitioner is guilty beyond reasonable doubt of violation of
Section 68 of PD 705.
16

Ruling:
No.
There are two distinct and separate offenses punished under Section 68 of PD
705, to wit:
(1)

Cutting, gathering, collecting and removing timber or other forest


products from any forest land, or timber from alienable or disposable
public land, or from private land without any authority; and

(2)

Possession of timber or other forest products without the legal


documents required under existing forest laws and regulations. [13]

The provision clearly punishes anyone who shall cut, gather, collect or remove timber or
other forest products from any forest land, or timber from alienable or disposable public
land, or from private land, without any authority. In this case, petitioner was charged by
CENRO to supervise the implementation of the permit. He was not the one who cut,
gathered, collected or removed the pine trees within the contemplation of Section 68 of
PD 705. He was not in possession of the cut trees because the lumber was used by
Teachers Camp for repairs. Petitioner could not likewise be convicted of conspiracy to
commit the offense because all his co-accused were acquitted of the charges against
them.
Petitioner may have been remiss in his duties when he failed to restrain the
sawyers from cutting trees more than what was covered by the permit. As the Court of
Appeals ruled, petitioner could have informed his superiors if he was really intimidated
by Santiago. If at all, this could only make petitioner administratively liable for his
acts. It is not enough to convict him under Section 68 of PD 705.
Neither could petitioner be liable under the last paragraph of Section 68 of PD 705
as he is not an officer of a partnership, association, or corporation who ordered the
cutting, gathering, or collection, or is in possession of the pine trees.
Petitioner Ernesto Aquino is ACQUITTED of the charge of violation of Section 68
of Presidential Decree No. 705.

17

Daylinda A. Lagua, et al. VS. Hon. Vicente N. Cusi et al.,


160 SCRA 260
Facts:
On January 1, 1976, Atty. Ernesto Nombrado, legal counsel for defendants,
issued a memorandum to the Chief Security Guard of Defendant Eastcoast directing the
latter to prevent the passage of Plaintiff Laguas hauling trucks loaded with logs for the
Japanese vessel on the national highway loading towards where the vessel was
berthed. In compliance with this directive, the security force of Defendant Eastcoast
closed the road to the use by plaintiffs trucks and other equipments and effectively
prevented their passage thereof while the vehicles and trucks of other people were
curiously not disturbed and were allowed passage on the same road.
Upon representations made to Indalecio L. Aspiras, Acting Station Officer-inCharge, BFD Lambajon Forest Station, and in response to plaintiff Laguas complaint, a
letter dated 2 January 1976 was addressed by Aspiras to the Resident Manager of
Defendant Eastcoast with instructions to open and allow Plaintiff Laguas trucks and
machineries to pass that road closed to them (but not to others) by Defendant
Eastcoast. Accordingly, Sagrado Constantino, Resident Manager of Defendant
Eastcoast, issued an order to their Chief Security Guard for the latter to comply with the
Aspiras letter. When Plaintiffs Laguas were already resuming the hauling operations of
their logs towards the Japanese Vessel on January 3, 1976, again that same road, only
the day before ordered by the BFD to be opened for use and passage by plaintiffs, was
closed to them by Defendant Eastcoasts security men upon a radio message order of
Defendant Maglana.
Given no recourse in the face of the blatant and illegal closure of the road in
defiance of BFD orders to the contrary by the Defendant Eastcoast through the order of
Defendant Maglana, Plaintiff Laguas had to depart posthaste to Mati, Davao Oriental,
from Baganga where the shipment and the road closure were made, to seek the
assistance of the PC Command thereat.
Issue:
Whether or not Bureau of Forest Development has the power and authority not
only to regulate the use or blockade of logging roads but also to exclusively determine
the legality of a closure of such roads.
Ruling:
P.D. No. 705 upon which the respondent court based its order does not vest any
power in the Bureau of Forest Development to determine whether or not the closure of a
logging road is legal or illegal and to make such determination a pre-requisite before an
action for damages may be maintained. Moreover, the complaint instituted by the
petitioners is clearly for damages based on the alleged illegal closure of the logging
road. Whether or not such closure was illegal is a matter to be established on the part of
18

the petitioners and a matter to be disproved by the private respondents. This should
appropriately be threshed out in a judicial proceeding. It is beyond the power and
authority of the Bureau of Forest Development to determine the unlawful closure of a
passage way, much less award or deny the payment of damages based on such
closure. Not every activity inside a forest area is subject to the jurisdiction of the Bureau
of Forest Development.
The private respondents, in their memorandum filed with the respondent court,
alleged that the logs of petitioner Achanzar were cut down and removed outside of the
area granted to the latter under his Private Timber License No. 2 and therefore inside
the concession area of respondent companys Timber License Agreement. This,
apparently, was the reason why the respondent company denied to the petitioners the
use of the logging road. If we hold the respondents to their contention that the Bureau of
Forest Development has the power and authority not only to regulate the use or
blockade of logging roads but also to exclusively determine the legality of a closure of
such roads, why then did they take it upon themselves to initially close the disputed
logging road before taking up the matter with the Bureau and why did they close it again
notwithstanding the Bureaus order to open it after the petitioners had duly informed the
said Bureau of the closure? To use the Bureaus authority which the respondents
ignored to now defeat the courts jurisdiction would be totally unacceptable. The trial
court committed grave abuse of discretion in dismissing the complaint on the ground of
lack of jurisdiction over the subject matter.
The petition is hereby GRANTED. The questioned order of the respondent court
is SET ASIDE and this case is ordered remanded to the court of origin for trial on the
merits.

19

MUSTANG LUMBER INC. VS. COURT OF APPEALS


257 SCRA 430
Facts:
A consolidation of three cases. Petitioner is a domestic corporation engaged in a
lumber dealer registered with the Bureau of Forest Development. Respondents are
DENR Sec. Factoran and Atty. Robles of the Special Actions and Investigations Division
(SAID) of the DENR.
Acting based on an information, the SAID team went to the lumberyard of
petitioner and based on a search warrant, were able to execute an administrative
seizure of different kinds of lumber, to which the petitioner failed to produce upon
demand the documents such as corresponding certificate of lumber origin and auxiliary
invoices which shall prove the legitimacy of their source and origin. Robles then
submitted a memorandum report to Factoran, ordering the cancellation of petitioners
Dealers Permit, filing of criminal charges, and confiscation of the trucks and lumbers.
Lower court ruled in favor of respondents, stating that possession of lumber without
permit or authority is not a crime.
Issue:
Whether or not a lumber cannot be considered timber and that petitioner should
not be held for illegal logging under Sec. 68 of the Revised Forestry Code.
Ruling:
No. While PD 705 explicitly provides that timber is included in the term forest
products, the term lumber is found in paragraph (aa) of Section 3 which states that the
latter is a processed log or processed forest raw material. Clearly, the law uses the word
lumber in its plain and common usage, and in the absence of a legislative intent to the
contrary, it shall be interpreted as such. Hence, it is safe to conclude that the law makes
no distinction whether the forest product is processed or not. Therefore, Judge Teresita
Capulong committed grave abuse of discretion in dismissing the case.

20

TAN VS. DIRECTOR OF FORESTRY


290 SCRA 187
Facts:
The Bureau of Forestry issued notice advertising for public bidding a certain tract
of public forest land situated in Olangapo, Zambales. One of the bidders is petitioner
Tan who was later then awarded such lot. On April 22, 1963, Ordinary Timber License
No. 20-'64, in the name of Wenceslao Vinzons Tan, was signed by then Acting Director
of Forestry Estanislao R. Bernal without the approval of the Secretary of Agriculture and
Natural Resources. On January 6, 1964, the license was released by the Office of the
Director of Forestry. It was not signed by the Secretary of Agriculture and Natural
Resources as required by Order No. 60. And because of this, that Timber license was
declared void ab initio and directed to stop the logging operations of Wenceslao Vinzons
Tan. Petitioner averred that the respondents-appellees unlawfully, illegally whimsically,
capriciously and arbitrarily acted without or in excess of their jurisdiction, and/or with
grave abuse of discretion by revoking a valid and existing timber license without just
cause, by denying petitioner-appellant of the equal protection of the laws, by depriving
him of his constitutional right to property without due process of law, and in effect, by
impairing the obligation of contracts.
Issue:
Whether or not the timber license was valid or not.
Held:
No. Court fully concur with the findings of the trial court that petitioner- appellant's
timber license was signed and released without authority by then Acting Director
Estanislao R. Bernal of Forestry, and is therefore void ab initio. In the first place, in
general memorandum order No. 46 dated May 30, 1963, the Director of Forestry was
authorized to grant a new ordinary timber license only where the area covered thereby
was not more than 3,000 hectares; the tract of public forest awarded to the petitioner
contained 6,420. In the second place, at the time it was released to the petitioner, the
Acting Director of Forestry had no more authority to grant any license. However,
granting that the timber license was valid, still respondents-appellees can validly revoke
his timber license. As pointed out, the rules and regulations included in the ordinary
timber license states: "The terms and conditions of this license are subject to change at
the discretion of the Director of Forestry, and that this license may be made to expire at
an earlier date, when public interests so require". A timber license is an instrument by
which the State regulates the utilization and disposition of forest resources to the end
that public welfare is promoted. A timber license is not a contract within the purview of
the due process clause; it is only a license or privilege, which can be validly withdrawn
whenever dictated by public interest or public welfare as in this case.
21

TAOPA VS. PEOPLE OF THE PHILIPPINES


571 SCRA 117
Facts:
The Community Environment and Natural Resources Office of Virac,
Catanduanes seized a truck loaded with illegally-cut lumber (113 pieces of lumber of
Philippine Mahogany Group and Apitong species without any authority and/or legal
documents as required under existingforest laws and regulations, prejudicial to the
public interest) and arrested its driver, Placido Cuison. The lumber was covered with
bundles of abaca fiber to prevent detection. On investigation, Cuison pointed to
Petitioner Amado Taopa and a certain Rufino Ogalesco as the owners of the seized
lumber. Taopa, Ogalesco and Cuison were charged with violating Section 68 of
Presidential Decree(PD) No. 705 as amended, in the RTC Virac, Catanduanes. Taopa,
Ogalesco and Cuison pleaded not guilty on arraignment. After trial on the merits, RTC
found them guilty as charged beyond reasonable doubt. Only Taopa and Cuison
appealed to CA, Cuison was acquitted but Taopa's conviction was affirmed. The
dispositive portion of the CA decision read: In this petition, Taopa seeks his acquittal
from the charges against him alleging that the prosecution failed to prove that he was
one of the owners of the seized lumber as he was not in the truck when the lumber was
seized.
Issue:
Whether or not petitioner is guilty of violating Section 68 of PD No. 705
Held:
Both the RTC and the CA gave scant consideration to Taopas alibi because
Cuisons testimony proved Taopas active participation in the transport of the seized
lumber. In particular, the RTC and the CA found that the truck was loaded with the cargo
in front of Taopas house and that Taopa and Ogalesco were accompanying the truck
driven by Cuison up to where the truck and lumber were seized. These facts proved
Taopas (and Ogalescos) exercise of dominion and control over the lumber loaded in
the truck. The acts of Taopa (and of his co-accused Ogalesco) constituted possession of
timber or other forest products without the required legal documents. Moreover, the fact
that Taopa and Ogalesco ran away at the mere sight of the police was likewise largely
indicative of guilt.
We are thus convinced that Taopa and Ogalesco were owners of the seized
lumber. Section 68 of PD 705, as amended, refers to Articles 309 and 310 of the
Revised Penal Code (RPC) for the penalties to be imposed on violators. Violation of
Section 68 of PD 705, as amended, is punished as qualified theft. The law treats
cutting, gathering, collecting and possessing timber or other forest products without
license as an offense as grave as and equivalent to the felony of qualified theft. The
22

actual market value of the 113 pieces of seized lumber was P67,630. Following Article
310 in relation to Article 309, the imposable penalty should be reclusion temporal in its
medium and maximum periods or a period ranging from 14 years, eight months and one
day to 20 years plus an additional period of four years for the excess of P47,630. The
minimum term of the indeterminate sentence imposable on Taopa shall be the penalty
next lower to that prescribed in the RPC. In this case, the minimum term shall be
anywhere between 10 years and one day to 14 years and eight months or prision mayor
in its maximum period to reclusion temporal in its minimum period.

23

PERFECTO PALLADA VS. PEOPLE OF THE PHILIPPINES


385 Phil. 195

Facts:
Sometime in the latter part of 1992, DENR received a reports that illegally cut lumber were
delivered in the warehouse of Valencia Golden Harvest Corporation in Valencia Bukidnon. DENR
officers in collaboration of PNP raided the companys warehouse and found a large stockpile of
lumber invarying sizes cut by a chainsaw.
As proof that the company had acquired the lumber by purchase, petitioner produced two receipts
issued by R.L. Rivero Lumberyard of Maramag, Bukidnon, dated March 6 and 17, 1992. The
DENR officers did not, however, give credit to the receipt considering that R. L. Rivero
Lumberyard's permit to operate had long been suspended. What is more, the pieces of lumber
were cut by chain saw and thus could not have come from a licensed sawmill operator. On
February 23, 1993, petitioner, as general manager, together with Noel Sy, as assistant operations
manager, and Francisco Tankiko, as president of the Valencia Golden Harvest Corporation, and
Isaias Valdehueza, were charged with violation of section 68 of P.D. No. 705, as amended.
During the trial, the accused presented documents that the lumber are legally
obtained. This may include the certificate of origin. However, the court found out that Pallada
was guilty of the violation of PD 705 and the rest of the accused were acquitted due to
insufficiency of evidence. The case was appealed to the CA and rendered a decision affirming
the decision of the lower court, thus this case was elevated.
Issue:
Whether or not a separate certificates of origin is used for lumber and timber.
Ruling:
Yes, there should be a separate Certificate of origin. The trial court acted correctly in
not giving credence to the Certificates of Timber Origin presented by petitioner since the
lumber held by the company should be covered by Certificates of Lumber Origin. For
indeed, as BFD Circular No. 10-83 states in pertinent parts:
In order to provide an effective mechanism to pinpoint accountability and
responsibility for shipment of lumber and to have uniformity in documenting
the origin thereof, the attached Certificate of Lumber.
Original CLO which form part of this circular is hereby adopted as
accountable forms for official use by authorized BFD officers
5. Lumber transported/shipped without the necessary Certificate of
Lumber Origin (CLO)as herein required shall be considered
as proceeding from illegal sources and as such, shall be subject to
24

confiscation and disposition in accordance with LOI 1020 and BFD


implementing guidelines.
The irregularities and discrepancies make the documents in which they are found not
only questionable but invalid and, thus, justified the trial court in giving no credence to the
same. The presence of such glaring irregularities negates the presumption that the CTOs were
regularly executed by the DENR officials concerned.

25

DAGUDAG VS. PADERANGA


555 SCRA 117
Facts:
On or about January 30, 2005, the Region VII Philippine National Police Regional
Maritime Group (PNPRMG) received information that MV General Ricarte of NMC
Container Lines, Inc. was shipping container vans containing illegal forest products from
Cagayan de Oro to Cebu. The shipments were falsely declared as cassava meal and
corn grains to avoid inspection by the Department of Environment and Natural
Resources (DENR).
On 30 and 31 January 2005, a team composed of representatives from the
PNPRMG, DENR, and the Philippine Coast Guard inspected the container vans at a
port in Mandaue City, Cebu. The team discovered the undocumented forest products
and the names of the shippers and consignees.
The crew of MV General Ricarte failed to produce the certificate of origin forms
and other pertinent transport documents covering the forest products, as required by
DENR Administrative Order No. 07-94.
Gen. Dagudag alleged that, since nobody claimed the forest products within a
reasonable period of time, the DENR considered them as abandoned and, on January
31, 2005, the Provincial Environment and Natural Resources Office (PENRO) Officer-inCharge (OIC), Richard N. Abella, issued a seizure receipt to NMC Container Lines, Inc.
On February 1, 2005, Community Environment and Natural Resources Office (CENRO)
OIC Loreto A. Rivac (Rivac) sent a notice to NMC Container Lines, Inc. asking for
explanation why the government should not confiscate the forest products.
In an affidavit dated 9 February 2005, NMC Container Lines, Inc.s Branch
Manager Alex Conrad M. Seno stated that he did not see any reason why the
government should not confiscate the forest products and that NMC Container Lines,
Inc. had no knowledge of the actual content of the container vans.
On 2, 9, and 15 February 2005, DENR Forest Protection Officer Lucio S. Canete,
Jr. posted notices on the CENRO and PENRO bulletin boards and at the NMC
Container Lines, Inc. building informing the unknown owner about the administrative
adjudication scheduled on 18 February 2005 at the Cebu City CENRO. Nobody
appeared during the adjudication.
In a resolution dated 10 March 2005, Rivac, acting as adjudication officer,
recommended to DENR Regional Executive Director Clarence L. Baguilat that the forest
products be confiscated in favor of the government.
In a complaint dated 16 March 2005 and filed before Judge Paderanga, a certain
Roger C. Edma (Edma) prayed that a writ of replevin be issued ordering the defendants
DENR, CENRO, Gen. Dagudag, and others to deliver the forest products to him and
that judgment be rendered ordering the defendants to pay him moral damages,
attorneys fees, and litigation expenses.
On 29 March 2005, Judge Paderanga issued a writ of replevin ordering Sheriff
Reynaldo L. Salceda to take possession of the forest products.

26

In a motion to quash the writ of replevin, the defendants DENR, CENRO, and
Gen. Dagudag prayed that the writ of replevin be set aside. Thereafter, Judge
Paderanga denied the motion to quash the writ of replevin for lack of merit.
Gen. Dagudag filed with the Office of the Court Administrator (OCA) an
affidavit-complaint dated July 8, 2005 charging Judge Paderanga with gross ignorance
of the law and conduct unbecoming a judge.
Issue:
Whether or not Judge Paderanga is liable for gross ignorance of the law and
conduct unbecoming a judge.
Ruling:
The Court finds Judge Paderanga liable for gross ignorance of the law and for
conduct unbecoming a judge.
The DENR is the agency responsible for the enforcement of forestry laws.
Section 4 of Executive Order No. 192 states that the DENR shall be the primary agency
responsible for the conservation, management, development, and proper use of the
countrys natural resources.
Section 68 of Presidential Decree No. 705, as amended by Executive Order No.
277, states that possessing forest products without the required legal documents is
punishable. Section 68-A states that the DENR Secretary or his duly authorized
representatives may order the confiscation of any forest product illegally cut, gathered,
removed, possessed, or abandoned.
In the instant case, the forest products were possessed by NMC Container Lines,
Inc. without the required legal documents and were abandoned by the unknown owner.
Consequently, the DENR seized the forest products.
Judge Paderanga should have dismissed the replevin suit outright for three
reasons:
First, under the doctrine of exhaustion of administrative remedies, courts cannot
take cognizance of cases pending before administrative agencies.In the instant case,
Edma did not resort to, or avail of, any administrative remedy. He went straight to court
and filed a complaint for replevin and damages. Section 8 of Presidential Decree No.
705, as amended, states that (1) all actions and decisions of the Bureau of Forest
Development Director are subject to review by the DENR Secretary; (2) the decisions of
the DENR Secretary are appealable to the President; and (3) courts cannot review the
decisions of the DENR Secretary except through a special civil action for certiorari or
prohibition.
Second, under the doctrine of primary jurisdiction, courts cannot take cognizance
of cases pending before administrative agencies of special competence. The DENR is
the agency responsible for the enforcement of forestry laws. The complaint for replevin
itself stated that members of DENRs Task Force Sagip Kalikasan took over the forest
products and brought them to the DENR Community Environment and Natural
Resources Office. This should have alerted Judge Paderanga that the DENR had
27

custody of the forest products, that administrative proceedings may have been
commenced, and that the replevin suit had to be dismissed outright.
Third, the forest products are already in custodia legis and thus cannot be the
subject of replevin. There was a violation of the Revised Forestry Code and the DENR
seized the forest products in accordance with law.
Judge Paderangas acts of taking cognizance of the replevin suit and of issuing the writ
of replevin constitute gross ignorance of the law.

28

FACTORAN V. COURT OF APPEALS


320 SCRA 530

Facts:

On August 9, 1988, 2 police officers of the Marikina intercepted a six-wheeler


truck, carrying narra lumber as it was cruising along the Marcos Highway. They
apprehended the truck driver, private respondent Jesus Sy, and brought the truck and
its cargo to the Personnel Investigation Committee/Special Actions and Investigation
Division (PIC/SAID) of the DENR Office in Quezon City. There, petitioner Atty. Vicente
Robles of the PIC/SAID investigated them, and discovered the discrepancies in the
documentation of the narra lumber. Due to the failure of respondents to show the
required documents, petitioner Factoran, then Secretary of Environment and Natural
Resources issued an order for the confiscation of the narra lumber and the six-wheeler
truck. Private respondents neither asked for reconsideration of nor appealed, the said
order to the Office of the President. Consequently, these items were then forfeited in
favor of the government. They were subsequently advertised to be sold at public
auction on March 20, 1989.
Respondents then filed for preliminary injunction and replevin, to which the trial
court acceded. Petitioner then refused to obey the writ of seizure and filed a
counterbond, to which the court denied because of lack of service to the respondents.
Court of appeals affirmed the lower courts decision.

Issue:

Whether or not the respondents can validly be restored possession of their trucks
and lumber based on the writ of replevin.

Ruling:

29

No. The herein respondents never appealed the confiscation order of petitioner
Secretary to the Office of the President as provided for in Sec. 8 of P.D. No. 705.
The doctrine of exhaustion of administrative remedies is basic. Courts, for reasons of
law, comity and convenience, should not entertain suits unless the available
administrative remedies have first been resorted to and the proper authorities have
been given an appropriate opportunity to act and correct their alleged errors, if any,
committed in the administrative forum. However, petitioners waived this ground for
failure to raise such in their motion to dismiss. Nevertheless, in order for replevin to
prosper, the wrongful detention by the defendant of the properties sought in an action
for replevin must be satisfactorily established. If only a mechanistic averment thereof is
offered, the writ should not be issued. In the case at bar, the subject narra lumber and
six-wheeler truck were confiscated by petitioner Secretary pursuant to Section 68-A of
P.D. No. 705, as amended by Executive Order (E.O.) No. 277.
Property lawfully taken by virtue of legal process is deemed to be in custodia
legis. When a thing is in official custody of a judicial or executive officer in pursuance of
his execution of a legal writ, replevin will not lie to recover it. Otherwise, there would be
interference with the possession before the function of law had been performed as to
the process under which the property was taken. Lastly, Sec. 80 of P. D. No. 705 which
requires delivery of the seized forest products within 6 hours from the time of the
seizure to the appropriate official designated by law to conduct preliminary
investigations applies only to criminal prosecutions provided for in Sec. 68, and not to
administrative confiscation provided for in Section 68-A.

30

PAAT VS. COURT OF APPEALS


266 SCRA 167
Facts:
On May 19, 1989, the truck of private respondent Victoria de Guzman while on
its way to Bulacan from Cagayan, was seized by DENR personnel in Nueva Vizcaya
because the driver could not produce the required documents for the forest products
found concealed in the truck. Petitioner Jovito Layugan, the Community Environment
and Natural Resources Officer (CENRO) in Aritao, Cagayan, issued an order of
confiscation of the truck and gave the owner 15 days within which to submit an
explanation why the truck should not be forfeited. Private respondents, however, failed
to submit the required explanation. Later, the Regional Executive Director of DENR
sustained petitioner Layugans action of confiscation and ordered the forfeiture of the
truck invoking Section 68-A of Presidential Decree No. 705 as amended by Executive
Order No. 277. Respondents then appealed.
Pending resolution however of the appeal, a suit for replevin was filed by the
private respondents against petitioner Layugan and Executive Director, which thereafter
issued a writ ordering the return of the truck to private respondents. Petitioner Layugan
and Executive Director Baggayan filed a motion to dismiss with the trial court
contending, inter alia, that private respondents had no cause of action for their failure to
exhaust administrative remedies. The trial court denied the motion to dismiss, which the
CA affirmed upon petitioners appeal.

Issues:

31

1. Whether or not an action for replevin prosper to recover a movable property


which is the subject matter of an administrative forfeiture proceeding in the
DENR pursuant to Section 68-A of P. D. 705.
2. Whether or not the Secretary of DENR and his representatives empowered to
confiscate and forfeit conveyances used in transporting illegal forest products in
favor of the government.

Ruling:

1.No, the Court held that before a party is allowed to seek the intervention of the
court, it is a pre-condition that he should have availed of all the means of administrative
processes afforded him. The premature invocation of courts intervention is fatal to ones
cause of action. In the case at bar, there is no question that the controversy was
pending before the Secretary of DENR when it was forwarded to him following the
denial by the petitioners of the motion for reconsideration of private respondents
through the order of July 12, 1989. In their letter of reconsideration dated June 28,
1989, private respondents clearly recognize the presence of an administrative forum to
which they seek to avail, as they did avail, in the resolution of their case.
2. Yes, as to the power of the DENR to confiscate, SECTION 68-A.
Administrative Authority of the Department or His Duly Authorized Representative To
Order Confiscation. In all cases of violation of this Code or other forest laws, rules and
regulations, the Department Head or his duly authorized representative, may order the
confiscation of any forest products illegally cut, gathered, removed, or possessed or
abandoned, and all conveyances used either by land, water or air in the commission of
the offense and to dispose of the same in accordance with pertinent laws, regulations
and policies on the matter.
It is, thus, clear from the foregoing provision that the Secretary and his duly
authorized representatives are given the authority to confiscate and forfeit any
conveyances utilized in violating the Code or other forest laws, rules and regulations.
Lastly, as to the contention that since they are not liable for qualified theft, then
they should not have necessarily have committed a crime under Sec. 68. This is
unmeritorious. With the introduction of Executive Order No. 277 amending Section 68 of
P.D. 705, the act of cutting, gathering, collecting, removing, or possessing forest
products without authority constitutes a distinct offense independent now from the crime
of theft under Articles 309 and 310 of the Revised Penal Code, but the penalty to be
imposed is that provided for under Article 309 and 310 of the Revised Penal Code. This
32

is clear from the language of Executive Order No. 277 when it eliminated the phrase
shall be guilty of qualified theft as defined and punished under Articles 309 and 310 of
the Revised Penal Code and inserted the words shall be punished with the penalties
imposed under Article 309 and 310 of the Revised Penal Code .

ALVAREZ V. PICOP
508 SCRA 498
Facts:
PICOP (Paper Industries Corp of the Phil) filed with the DENR an application to
have its Timber License Agreement (TLA) No. 43 converted into an IFMA (Integrated
forest management agreement). In the middle of the processing of PICOPs application,
however, PICOP refused to attend further meetings with the DENR. Instead, on 2
September 2002, PICOP filed before the RTC of Quezon City a Petition for Mandamus 1
against then DENR Secretary Heherson T. Alvarez. PICOP seeks the issuance of a
privileged writ of mandamus to compel the DENR Secretary to sign, execute and deliver
an IFMA to PICOP. Lower court ruled in favor of respondents, CA affirmed, but upon
appeal to the Supreme Court, it reversed the judgment of the CA. Now PICOP files a
Motion for Reconsideration.
Issue:

33

Whether or not PICOP is entitled to the IFMA by way of mandamus and due to
the non-impairment clause of the Constitution in relation to Document 1969.
Ruling:
NO, Document 1969 is not a contact, but a mere collateral undertaking pursuant
to the TLA.
An examination of the Presidential Warranty (Doc. 1969) at once reveals that it
simply reassures PICOP of the governments commitment to uphold the terms and
conditions of its timber license and guarantees PICOPs peaceful and adequate
possession and enjoyment of the areas which are the basic sources of raw materials for
its wood processing complex. The warranty covers only the right to cut, collect, and
remove timber in its concession area, and does not extend to the utilization of other
resources, such as mineral resources, occurring within the concession. The Presidential
Warranty cannot be considered a contract distinct from PTLA No. 47 and FMA No. 35. It
is merely a collateral undertaking which cannot amplify PICOPs rights under its timber
license. The ruling in Oposa v. Factoran that a timber license is not a contract within the
purview of the non-impairment clause is edifying.
Also, PICOP failed to secure NCIP Cerification based on R.A. 8371 and
Sanggunian consultation and approval for environmentally critical projects based on the
Local Government Code.

ALVAREZ V. PICOP
606 SCRA 444

Facts:

This is a consolidation of three cases. On 23 December 1999, then DENR


Secretary Cerilles promulgated DENR Administrative Order (DAO) No. 99-53 which had
for its subject, the "Regulations Governing the Integrated Forest Management Program
(IFMP)."In a 28 August 2000 letter to the Community Environment and Natural
Resources Office (CENRO), PICOP signified its intention to convert its TLA No. 43 into
an Integrated Forest Management Agreement (IFMA) invoking the provisions of Section
9, Chapter III of DAO No. 99-53. Pursuant to the application for conversion, the
Performance Evaluation Team of DENR conducted a report indicating violations by
34

PICOP of existing DENR Rules and Regulations governing TLA No. 43, such as the
non-submission of its five-year forest protection plan and seven-year reforestation plan
as required by the DENR rules and regulations, as well as overdue and unpaid forest
charges. Upon submission of the memorandum to the DENR Secretary, negotiations
were held to assure PICOPs compliance with DAO 99-53, however PICOP later
claimed that the conversion had already been completed pursuant to Sec. Alvarez letter
of clearance.
PICOP then filed for a petition for mandamus against petitioner which was
granted by the trial court. Later on, Gozun was substituted in the case as the new
DENR Secretary. Court of Appeals eventually affirmed the lower courts ruling, to which
herein petitioners now appeal.

Issue:

Whether or not PICOP is entitled by way of mandamus to the conversion of its


TLA to IFMA, and WON there was compliance with DAO No. 99-53 and thus conversion
had already been effected.

Ruling:

No. A timber license agreement is not a contract, neither is the presidential


warranty of President Marcos a contract. The argument that the Presidential Warranty is
a contract on the ground that there were mutual considerations taken into account
consisting in investments on PICOPs part is preposterous. All licensees put up
investments in pursuing their businesses. To construe these investments as
consideration in a contract would be to stealthily render ineffective the settled
jurisprudence that "a license or a permit is not a contract between the sovereignty and
the licensee or permittee, and is not a property in the constitutional sense, as to which
the constitutional proscription against the impairment of contracts may extend."
As to the compliance with the DAO, the following are the requisites for the
automatic conversion of the TLA into an IFMA, to wit:
1. The TLA holder had signified its intent to convert its TLA into an IFMA prior to the
expiration of its TLA;
2. Proper evaluation was conducted on the application; and

35

3. The TLA holder has satisfactorily performed and complied with the terms and
conditions of the TLA and the pertinent rules and regulations.
In the case at bar, PICOP failed to comply with DAO, as evidenced by the
memorandum submitted to the DENR Secretary by the Performance Evaluation Team.
Finally, the DENR, by withholding the conversion of PICOPs TLA No. 43 into an IFMA,
has made a factual finding that PICOP has not yet complied with the requirements for
such a conversion. Findings of facts of administrative agencies are generally accorded
great respect, if not finality, by the courts because of the special knowledge and
expertise over matters falling under their jurisdiction.
Lastly, as to whether or not conversion already took place, the court ruled in the
negative. By giving this clearance for the conversion of PICOPs TLA into an IFMA, the
DENR Secretary cannot, by any stretch of imagination, be claimed to have granted the
conversion itself. The letter is clear that the "conversion" could not be final since its
conditions and details still have to be discussed as stated in the second paragraph of
said letter; hence, the same letter could not have reduced to a mere formality the
approval of the conversion of PICOPs TLA No. 43 into an IFMA. Even assuming,
however, that the IFMA has already been converted, this is all purely academic because
of the above-discussed settled jurisprudence that logging permits are not contracts
within the Non-Impairment Clause and thus, can be amended, modified, replaced or
rescinded when the national interest so requires. If the DENR Secretary, therefore, finds
that the IFMA would be in violation of statutes, rules and regulations, particularly those
protecting the rights of the local governments and the indigenous peoples within the
IFMA area, then it behooves the DENR Secretary to revoke such IFMA. These same
statutes, rules and regulations are the very same requirements mentioned above for the
conversion of the TLA No. 43 into an IFMA.

36

MATUGUINA INTEGRATED WOOD PRODUCTS, INC. VS. COURT OF APPEALS


263 SCRA 490
Facts:
A license was issued in 1973 in favor of Matuguina Logging Enterprises. Davao
Enterprises Corporation filed a complaint against MLE alleging that MLE encroached on
the area allotted to DAVENCORs timber concession. The investigating committee of the
District Forester of Davao found MLE guilty. MLE appealed to the Ministry of Natural
Resources, which also found MLE guilty. DAVENCOR then requested that MLE be
directed to comply with the order and pay the value of the 2,352.04 cubic meters of
timber. The rights of MLE were then transferred to MIWPI as requested by Milagros
Matuguina who is a majority stockholder of MLE.
Issue:
Whether or not MIWPI can be held liable for MLEs encroachment on
DAVENCORs timber concession
Held:
No, although under PD 705 the transferee assumes all the obligations of the
transferor nevertheless the obligations referred to under Sec.61 of PD 705 are those
incurred by the transferor in the ordinary course of business and not those incurred by
transferor as a result of transgressions of the law, for these are considered personal
obligations of the transferor and cannot be included in the obligations to be passed on
to the transferee absent any modifying provisions to that effect. Therefore, MIWPI
cannot be held liable for the encroachment on DAVENCORs concession.

37

DY VS. COURT OF APPEALS


304 SCRA 331

Facts:

On May 31, 1993, the Mayor of Butuan City issued Executive Order No. 93-01
creating Task Force Kalikasan to combat illegal logging, log smuggling or possession of
and/or transport of illegally cut or produced logs, lumber, flitches and other forest
products in that city. The team was composed of personnel of the Philippine Army,
PNP, DENR and the Office of the City Mayor of Butuan. Respondent Odel Bernardo
Lausa, who was the acting chief of civilian security in the mayors office, was a member
of the team. On July 1, 1993, the members of the task force received confidential
information that two truckloads of illegally cut lumber would be brought to Butuan City
from the Ampayon-Taguibe-Tiniwisan area. Accordingly, the team set up a checkpoint
along kilometer 4 in Baan, Butuan City. Therafter upon catching up with the two cars in
the latters compound, the caretaker of the compound was not able to produce any
documents proving the legality of possession of the forest products. DENR officers then
seized the truck and lumber, and since there were no claimants after posting the notice
of confiscation, it was deemed forfeited in favor of the government.
2 months after the said forfeiture, petitioner herein filed a suit for replevin to
recover the trucks and lumber, to which respondent Lausa filed a motion for approval of
counterbond and dismissal of the replevin since the seizure was pursuant to the
38

Revised Forestry Code. Trial court for petitioner, but was reversed in the CA in favor of
Lausa. Hence this petition.

Issue:

Whether or not the Regional Trial Court could in fact take cognizance of the
replevin suit, considering that the object was the recovery of lumber seized and forfeited
by law enforcement agents of the DENR pursuant to P.D. No. 705 (Revised Forestry
Code), as amended by Executive Order No. 277.

Ruling:

No. The rule is that a party must exhaust all administrative remedies before he
can resort to the courts. In a long line of cases, we have consistently held that before a
party may be allowed to seek the intervention of the court, it is a pre-condition that he
should have availed himself of all the means afforded by the administrative processes.
Hence, if a remedy within the administrative machinery can still be resorted to by giving
the administrative officer concerned every opportunity to decide on a matter that comes
within his jurisdiction then such remedy should be exhausted first before a courts
judicial power can be sought. The premature invocation of a courts intervention is fatal
to ones cause of action. Accordingly, absent any finding of waiver or estoppel, the case
is susceptible of dismissal for lack of cause of action.
As petitioner clearly failed to exhaust available administrative remedies, the
Court of Appeals correctly set aside the assailed orders of the trial court granting
petitioners application for a replevin writ and denying private respondents motion to
dismiss. Having been forfeited pursuant to P.D. No. 705, as amended, the lumber
properly came under the custody of the DENR and all actions seeking to recover
possession thereof should be directed to that agency.
The appellate courts directive to the trial court judge to allow the respondent
agent of the DENR to file a counterbond in order to recover custody of the lumber
should be disregarded as being contrary to its order to dismiss the replevin suit of
petitioner. For, indeed, what it should have done was to dismiss the case without
prejudice to petitioner filing her claim before the DENR .

39

PICOP RESOURCES VS. BASE METALS


510 SCRA 400

Facts:

In 1987, the Central Mindanao Mining and Development Corporation (CMMCI for
brevity) entered into a Mines Operating Agreement with Banahaw Mining and
Development Corporation whereby the latter agreed to act as Mine Operator for the
40

exploration, development, and eventual commercial operation of CMMCI's 18 mining


claims located in Agusan del Sur. Pursuant to the terms of the Agreement, Banahaw
Mining filed applications for Mining Lease Contracts over the mining claims with the
Bureau of Mines. On April 29, 1988, Banahaw Mining was issued a Mines Temporary
Permit authorizing it to extract and dispose of precious minerals found within its mining
claims. Since a portion of Banahaw Mining's mining claims was located in petitioner
PICOP's logging concession in Agusan del Sur, Banahaw Mining and petitioner PICOP
entered into a MOA whereby petitioner PICOP allowed Banahaw Mining an access to its
mining claims. In 1991, Banahaw Mining converted its mining claims to applications for
Mineral Production Sharing Agreements (MPSA for brevity).
While the MPSA were pending, Banahaw Mining, on December 18, 1996,
decided to sell/assign its rights and interests over 37 mining claims in favor of private
respondent Base Metals Mineral Resources Corporation. The transfer included those
covered by its mining operating agreement with CMMCI. Upon being informed of the
development, CMMCI, as claim owner, immediately approved the assignment made by
Banahaw Mining in favor of private respondent Base Metals, thereby recognizing private
respondent Base Metals as the new operator of its claims. On March 10, 1997, private
respondent Base Metals amended Banahaw Mining's pending MPSA applications with
the Bureau of Mines to substitute itself as applicant and to submit additional documents
in support of the application. Area clearances from the DENR Regional Director and
Superintendent of the Agusan Marsh and Wildlife Sanctuary were submitted, as
required.
On November 18, 1997, petitioner PICOP filed with the Mines Geo-Sciences
Bureau (MGB), an Opposition to private respondent Base Metals' application because it
violate the non-impairment clause and will be prejudicial to herein petitioner. The Panel
Arbitrator initially ruled for petitioner, but upon appeal to the Mines Adjudication Board,
judgment was in favor of respondent, CA affirmed stating that the Presidential Warranty
of September 25, 1968 issued by then President Ferdinand E. Marcos merely confirmed
the timber license granted to PICOP and warranted the latter's peaceful and adequate
possession and enjoyment of its concession areas. It was only given upon the request
of the Board of Investments to establish the boundaries of PICOP's timber license
agreement. The Presidential Warranty did not convert PICOP's timber license into a
contract because it did not create any obligation on the part of the government in favor
of PICOP. Thus, the non-impairment clause finds no application.

Issue:
Whether or not the concession area of petitioner is closed to mining activities and
that the conversion of the agreement into MPSA will run counter to the non-impairment
clause of the Constitution.
Ruling:

41

NO, a Timber license agreement is not a contract, but a mere privilege.


We should state at this juncture that the policy of multiple land use is enshrined in our
laws towards the end that the country's natural resources may be rationally explored,
developed, utilized and conserved. In like manner, RA 7942, recognizing the
equiponderance between mining and timber rights, gives a mining contractor the right to
enter a timber concession and cut timber therein provided that the surface owner or
concessionaire shall be properly compensated for any damage done to the property as
a consequence of mining operations.
Firstly, assuming that the area covered by Base Metals' MPSA is a government
reservation, defined as proclaimed reserved lands for specific purposes other than
mineral reservations, such does not necessarily preclude mining activities in the area.
Sec. 15(b) of DAO 96-40 provides that government reservations may be opened for
mining applications upon prior written clearance by the government agency having
jurisdiction over such reservation. Sec. 6 of RA 7942 also provides that mining
operations in reserved lands other than mineral reservations may be undertaken by the
DENR, subject to certain limitations.
Secondly, RA 7942 does not disallow mining applications in all forest reserves
but only those proclaimed as watershed forest reserves. There is no evidence in this
case that the area covered by Base Metals' MPSA has been proclaimed as watershed
forest reserves. DENR Memorandum Order No. 03-98, which provides the guidelines in
the issuance of area status and clearance or consent for mining applications pursuant to
RA 7942, provides that timber or forest lands, military and other government
reservations, forest reservations, forest reserves other than critical watershed forest
reserves, and existing DENR Project Areas within timber or forest lands, reservations
and reserves, among others, are open to mining applications subject to area status and
clearance.
Lastly, PICOP failed to present any evidence that the area covered by the MPSA
is a protected wilderness area designated as an initial component of the NIPAS
pursuant to a law, presidential decree, presidential proclamation or executive order as
required by RA 7586.

ASAPHIL VS. TUASON


488 SCRA 126
Facts:

42

On March 24, 1975, respondent Vicente Tuason, Jr. entered into a Contract for
Sale and Purchase of Perlite Ore with Induplex wherein Induplex agreed to buy all the
perlite ore that may be found and mined in Tuasons mining claim located in Taysa,
Daraga, Albay. In exchange, Induplex will assist Tuason in securing and perfecting his
right over the mining claim . Thereafter, Tuason executed an Agreement to Operate
Mining Claims in favor of petitioner Asaphil Construction and Development Corporation.
Later, Tuason filed with the Bureau of Mines, DENR a complaint against Asaphil and
Induplex for declaration of nullity of the said Contracts. Tuason alleged in his complaint
that the stockholders of Induplex formed and organized Ibalon Mineral Resources, an
entity whose purpose is to mine any and all kinds of minerals, that this is in violation of
the condition imposed by the Board of on Induplex in its Joint Venture Agreement with
Grefco, Inc, prohibiting Induplex from mining perlite ore, through an operating
agreement or any other method; that Induplex acquired the majority stocks of Asaphil
and that 95% of Ibalons shares were also transferred to Virgilio R. Romero, who is a
stockholder of Induplex, Asaphil and Ibalon. Tuason claimed that said acts adversely
affected, not only his interest as claimowner, but the governments interest as well.
Asaphil filed its Answer, praying for the dismissal of the complaint on the ground
that the DENR has no jurisdiction over the case. Induplex filed a Motion to Dismiss the
complaint, also on ground of lack of jurisdiction. Induplex contended that to fall within
the jurisdiction of the DENR, the controversy should involve a mining property and the
contending parties must be claimholders and/or mining operators; and that the dispute
in this case involves mineral product and not a mining property, and the protagonists
are claimholders (Tuason) and a buyer (Induplex). DENR affirmed, but the Mines
Adjucation Board reversed, stating that the complaint is for the cancellation and
revocation of the Agreement to Operate Mining Claims, which is within the jurisdiction of
the DENR under Section 7 of PD 1281. The MAB also found that the acquisition by
Induplex of the majority stocks of Asaphil, and Induplexs assumption of the mining
operation violated the BOI prohibition.
Issue:
Whether or not the DENR has jurisdiction over Tuasons complaint for the
annulment of the Contract for Sale and Purchase of Perlite Ore between Tuason and
Induplex, and the Agreement to Operate Mining Claims between Tuason and Asaphil;
and second, WON the MAB erred in invalidating the Agreement to Operate Mining
Claims.

Ruling:

43

Yes. In several cases on mining disputes, the Court recognized a distinction


between (1) the primary powers granted by pertinent provisions of law to the then
Secretary of Agriculture and Natural Resources (and the bureau directors) of an
executive or administrative nature, such as granting of license, permits, lease and
contracts, or approving, rejecting, reinstating or canceling applications, or deciding
conflicting applications, and (2) controversies or disagreements of civil or contractual
nature between litigants which are questions of a judicial nature that may be adjudicated
only by the courts of justice.The allegations in Tuasons complaint do not make out a
case for a mining dispute or controversy within the jurisdiction of the DENR. While the
Agreement to Operate Mining Claims is a mining contract, the ground upon which the
contract is sought to be annulled is not due to Asaphils refusal to abide by the terms
and conditions of the agreement, but due to Induplexs alleged violation of the condition
imposed by the BOI in its Joint Venture Agreement with Grefco, Inc.. Also, Tuason
sought the nullity of the Contract for Sale and Purchase of Perlite Ore, based on the
same alleged violation. Obviously, this raises a judicial question, which is proper for
determination by the regular courts.
The DENR is not called upon to exercise its technical knowledge or expertise
over any mining operations or dispute; rather, it is being asked to determine the validity
of the agreements based on circumstances beyond the respective rights of the parties
under the two contracts. Thus, the DENR Regional Executive Director was correct in
dismissing the complaint for lack of jurisdiction over Tuasons complaint; consequently,
the MAB committed an error in taking cognizance of the appeal, and in ruling upon the
validity of the contracts.

DIDIPIO EARTH SAVERS MULTI-PURPOSE ASSOCIATION VS. GOZUN


44

485 SCRA 586

Facts:

In 1987, Cory promulgated EO 279 which empowered DENR to stipulate with


foreign companies regarding technical or financial large scale exploration or mining. In
1995, Ramos signed into law RA 7942 or the Philippine Mining Act. In 1994, Ramos
already signed an FTAA with Arimco Mining Co, an Australian company. The FTAA
authorized AMC (later CAMC) to explore 37,000 hectares of land in Quirino and Nueva
Vizcaya including Brgy .Didipio. After the passage of the law, DENR then issued its
Implementing Rules and Regulations.
In seeking to nullify Rep. Act No. 7942 and DAO 96-40 as unconstitutional,
petitioners reasoned that these in effect allow the unlawful and unjust taking of
property for private purpose in contravention with Section 9, Article III of the 1987
Constitution, mandating that private property shall not be taken except for public use
and with the corresponding payment of just compensation. They assert that public
respondent DENR, through the Mining Act and its Implementing Rules and Regulations,
cannot, on its own, permit entry into a private property and allow taking of land without
payment of just compensation.
Public respondents on the other hand avers that Section 76 is not a taking
provision but a valid exercise of the police power and by virtue of which, the state may
prescribe regulations to promote the health, morals, peace, education, good order,
safety and general welfare of the people. This government regulation involves the
adjustment of rights for the public good and it endeavors potential for the use or
economic exploitation of private property. Public respondents concluded that to require
compensation in all such circumstances would compel the government to regulate by
purchase.
Issue:
Whether or not RA 7942 and the DENR RRs are valid.
Ruling:
Yes. The SC noted the requisites of eminent domain. They are following:
(1) the expropriator must enter a private property;
(2) the entry must be for more than a momentary period.
(3) the entry must be under warrant or color of legal authority;
(4)
the property must be devoted to public use or otherwise informally
appropriated or injuriously affected;
45

(5) the utilization of the property for public use must be in such a way as to oust the
owner and deprive him of beneficial enjoyment of the property.
In the case at bar, Didipio failed to show that the law is invalid. Indeed there is
taking involved but it is with just compensation. Sec. 76 of RA 7942 provides for just
compensation as well as section 107 of the DENR RR. To wit:
Section 76. xxx Provided, that any damage to the property of the surface owner,
occupant, or concessionaire as a consequence of such operations shall be
properly compensated as may be provided for in the implementing rules and
regulations.
Section 107. Compensation of the Surface Owner and Occupant- Any
damage done to the property of the surface owners, occupant, or concessionaire
thereof as a consequence of the mining operations or as a result of the
construction or installation of the infrastructure mentioned in 104 above shall be
properly and justly compensated.
Further, mining is a public policy and the government can invoke eminent domain to
exercise entry, acquisition and use of private lands.

46

REPUBLIC VS. ROSEMOOR


421 SCRA 148
Facts:
Rosemoor Mining And Development Corporation Corporation (Rosemoor), after
having been granted permission to prospect for marble deposits in the mountains of
Biak-na-Bato, Bulacan, succeeded in discovering marble deposits of high quality and in
commercial quantities. They applied with the Bureau of Mines (now Mines and
Geosciences Bureau), for the issuance of the corresponding license to exploit said
marble deposits, which was issued to them, giving them the right to quarry 330 hectares
of land. After Ernesto Maceda was appointed Minister of DENR, he cancelled
Rosemoors license. Rosemoor filed for injunctive relief from the RTC, which ruled in
their favor.
The RTC said that Rosewoods respondents license had already ripened into a
property right, which was protected under the due process clause, and such right was
supposedly violated when the license was unjustifiably cancelled without notice and
hearing. Petitioners aver that the license contravenes PD 463 because it exceeds the
maximum area that may be granted to a Licensee for quarrying (100 hectares), which
renders the license void. The CA sustained the RTC decision because the license was
embraced by four (4) separate applications, and that the 100 hectare limitation was
superseded by RA 7942. The CA also said that Proclamation 84, which confirmed the
cancellation of the license, impaired the non-impairment clause of contracts, a bill of
attainder and an ex post facto law.
Issue:
Whether or not Rosemoors license was validly cancelled.
Ruling:
Yes.
Validity of License
PD 463, as amended, pertained to the old system of exploration, development
and utilization of natural resources through licenses, concessions or leases, but was
omitted in the 1987 Constitution as it was deemed violative of its provisions. This was
replaced by RA 7942 or the Philippine Mining Act of 1995 repealed or amended all laws,
executive orders, presidential decrees, rules and regulations -- or parts thereof -- that
are inconsistent with it.
While RA 7942 has expressly repealed provisions of mining laws that are
inconsistent with its own, it nonetheless respects previously issued valid and existing
licenses. In this case, the terms of Rosemoors license was subject to PD 463, the
47

existing law when it was granted. And under such law, it is clear that a license should
only cover 100 hectares without exceptions or consideration to the number of
applications. The intent of the law would be brazenly circumvented by ruling that a
license may cover an area exceeding the maximum by the mere expediency of filing
several applications. Such ruling would indirectly permit an act that is directly prohibited
by the law.
Validity of Proclamation No. 84
Rosemoors license may be revoked or rescinded by executive action when the
national interest so requires, because it is not a contract, property or a property right
protected by the due process clause of the Constitution. This condition to the license
was acknowledged by Rosemoor in its permit. Moreover, granting that Rosemoor
license is valid, it can still be validly revoked by the State in the exercise of police power.
The exercise of such power through Proclamation No. 84 is clearly in accord with the
regalia doctrine which reserves to the State ownership of all natural resources.
Proclamation 84 does not impair the non-impairment clause because the license
is not a contract. Even if the license were, it is settled that provisions of existing laws
and a reservation of police power are deemed read into it, because it concerns a
subject impressed with public welfare. PN 84 is also not a bill of attainder because the
declaration that the license was void is not a punishment. It is also not an ex post facto
law because the proclamation does not fall under any of the enumerated categories of
an ex post facto law. And an ex post facto law is limited in its scope only to matters
criminal in nature.

48

LA BUGAL BLAAN TRIBAL ASSOCIATION INC., ET AL. V. RAMOS


421 SCRA 148
Facts:
On July 25, 1987, then President Corazon C. Aquino issued Executive Order
(E.O.) No. 279 authorizing the DENR Secretary to accept, consider and evaluate
proposals from foreign-owned corporations or foreign investors for contracts or
agreements involving either technical or financial assistance for large-scale exploration,
development, and utilization of minerals, which, upon appropriate recommendation of
the Secretary, the President may execute with the foreign proponent.
On March 3, 1995, then President Fidel V. Ramos approved R.A. No. 7942 to
"govern the exploration, development, utilization and processing of all mineral
resources." R.A. No. 7942 defines the modes of mineral agreements for mining
operations, outlines the procedure for their filing and approval, assignment/transfer and
withdrawal, and fixes their terms. Similar provisions govern financial or technical
assistance agreements.
On April 9, 1995, 30 days following its publication on March 10, 1995 in Malaya
and Manila Times, two newspapers of general circulation, R.A. No. 7942 took effect.
Shortly before the effectivity of R.A. No. 7942, however, or on March 30, 1995, the
President entered into an FTAA with WMCP covering 99,387 hectares of land in South
Cotabato, Sultan Kudarat, Davao del Sur and North Cotabato.
On August 15, 1995, then DENR Secretary Victor O. Ramos issued DENR
Administrative Order (DAO) No. 95-23, s. 1995, otherwise known as the Implementing
Rules and Regulations of R.A. No. 7942. This was later repealed by DAO No. 96-40, s.
1996 which was adopted on December 20, 1996.
On January 10, 1997, counsels for petitioners sent a letter to the DENR
Secretary demanding that the DENR stop the implementation of R.A. No. 7942 and
DAO No. 96-40, giving the DENR fifteen days from receipt to act thereon. The DENR,
however, has yet to respond or act on petitioners' letter.
Petitioners claim that the DENR Secretary acted without or in excess of
jurisdiction.
They pray that the Court issue an order:
(a) Permanently enjoining respondents from acting on any application for Financial or
Technical Assistance Agreements;
(b) Declaring the Philippine Mining Act of 1995 or Republic Act No. 7942 as
unconstitutional and null and void;
(c) Declaring the Implementing Rules and Regulations of the Philippine Mining Act
contained in DENR Administrative Order No. 96-40 and all other similar administrative
issuances as unconstitutional and null and void; and
(d) Cancelling the Financial and Technical Assistance Agreement issued to Western
Mining Philippines, Inc. as unconstitutional, illegal and null and void.
49

In January 2001, MMC a publicly listed Australian mining and exploration


company sold its whole stake in WMCP to Sagittarius Mines, 60% of which is owned
by Filipinos while 40% of which is owned by Indophil Resources, an Australian
company. DENR approved the transfer and registration of the FTAA in Sagittarius name
but Lepanto Consolidated assailed the same. WMCP contends that the annulment of
the FTAA would violate a treaty between the Philippines and Australia which provides
for the protection of Australian investments.
Issue:
1. Whether or not the Philippine Mining Act is unconstitutional for allowing fully
foreign-owned corporations to exploit Philippine mineral resources.
2. Whether or not the FTAA between WMCP and the Pgilippines is a service
Ruling:
1. Yes. RA 7942 or the Philippine Mining Act of 1995 is unconstitutional for permitting fully
foreign owned corporations to exploit Philippine natural resources. Article XII Section 2
of the 1987 Constitution retained the Regalian doctrine which states that All lands of
the public domain, waters, minerals, coal, petroleum, and other minerals, coal,
petroleum, and other mineral oils, all forces of potential energy, fisheries, forests or
timber, wildlife, flora and fauna, and other natural resources are owned by the State.
The same section also states that, exploration and development and utilization of
natural resources shall be under the full control and supervision of the State.
Conspicuously absent in Section 2 is the provision in the 1935 and 1973
Constitutions authorizing the State to grant licenses, concessions, or leases for the
exploration, exploitation, development or utilization of natural resources. Y such
omission, the utilization of inalienable lands of public domain through license,
concession or lease is no longer allowed under the 1987 Constitution.
Under the concession system, the concessionaire makes a direct equity
investment for the purpose of exploiting a particular natural resource within a given
area. The concession amounts to complete control by the concessionaire over the
countrys natural resource, for it is given exclusive and plenary rights to exploit a
particular resource at the point of extraction.
The 1987 Constitution, moreover, has deleted the phrase management or other
forms of assistance in the 1973 Charter. The present Constitution now allows only
technical and financial assistance. The management or operation of mining activities
by foreign contractors, the primary feature of service contracts was precisely the evil the
drafters of the 1987 Constitution sought to avoid.
The constitutional provision allowing the President to enter into FTAAs is an
exception to the rule that participation in the nations natural resources is reserved
exclusively to Filipinos. Accordingly such provision must be construed strictly against
their enjoyment by non-Filipinos. Therefore RA 7942 is invalid insofar as said act
authorizes service contracts. Although the statute employs the phrase financial and
technical agreements in accordance with the 1987 Constitution, its pertinent provisions
50

actually treat these agreements as service contracts that grant beneficial ownership to
foreign contractors contrary to the fundamental law.
The underlying assumption in the provisions of the law is that the foreign
contractor manages the mineral resources just like the foreign contractor in a service
contract. By allowing foreign contractors to manage or operate all the aspects of the
mining operation, RA 7942 has in effect conveyed beneficial ownership over the nations
mineral resources to these contractors, leaving the State with nothing but bare title
thereto.
The same provisions, whether by design or inadvertence, permit a circumvention
of the constitutionally ordained 60-40% capitalization requirement for corporations or
associations engaged in the exploitation, development and utilization of Philippine
natural resources.
When parts of a statute are so mutually dependent and connected as conditions,
considerations, inducements or compensations for each other as to warrant a belief that
the legislature intended them as a whole, then if some parts are unconstitutional, all
provisions that are thus dependent, conditional or connected must fall with them.
Under Article XII Section 2 of the 1987 Charter, foreign owned corporations are
limited only to merely technical or financial assistance to the State for large scale
exploration, development and utilization of minerals, petroleum and other mineral oils.
2. Yes. The FTAA between WMCP and the Philippine government is likewise
unconstitutional since the agreement itself is a device contract.
Section 1.3 of the FTAA grants WMCP, a fully foreign owned corporation, the
exclusive right to explore, exploit, utilize and dispose of all minerals and by-products
that may be produced from the contract area. Section 1.2 of the same agreement
provides that WMCP shall provide all financing, technology, management, and
personnel necessary for the Mining Operations.
These contractual stipulations and related provisions in the FTAA taken together,
grant WMCP beneficial ownership over natural resources that properly belong to the
State and are intended for the benefit of its citizens. These stipulations are abhorrent to
the 1987 Constitution. They are precisely the vices that the fundamental law seeks to
avoid, the evils that it aims to suppress. Consequently, the contract from which they
spring must be struck down.

51

LA BUGAL BLAAN TRIBAL ASSOCIATION INC., ET AL. V. RAMOS


445 SCRA 1
Facts:
In the January Decision, the Court en banc promulgated its Decision, granting
the Petition and declaring the unconstitutionality of certain provisions of RA 7942
(Philippine Mining Act), DENR AO 96-40, as well as of the entire FTAA executed
between the government and Western Mining Corp. Philippines, an Australian
corporation. (WMCP). The Court said RA 7942 or the Philippine Mining Act of 1995 and
its implementing rules are unconstitutional for allowing service contracts now
prohibited by the 1987 Charter. The Court said FTAA is a service contract that grants
control or beneficial ownership over the nations mineral resources to foreign
contractors, leaving the State with nothing but bare title thereto. It was also on this
ground that the Court struck down as constitutionally infirm the FTAA between the
government and WMCP.
Issue:
Whether or not the phrase Agreements Involving Either Technical or Financial
Assistance contained in paragraph 4 of Section 2 of Article XII of the Constitution was
properly interpreted in the previous decision to warrant the unconstitutionality of RA
7942 and the FTAA of WMCP.
Ruling:
Yes.
The Proper Interpretation of the Constitutional Phrase Agreements
Involving either Technical or Financial Assistance
Section 2 Article XII does not reveal any intention to proscribe foreign
involvement in the management or operation of mining activities or to eliminate service
contracts, nor does it have express prohibition to this effect. Had the framers intended
to prohibit direct participation of an alien corporation in the exploration of the countrys
natural resources, they would employed clearly restrictive language barring foreign
corporation from directly engaging in the exploration of the countrys natural resources.
Foreign corporations may indeed participate in the exploitation, development and
use of Philippine natural resources but subject to the full control and supervision of the
State. RA 7942, its implementing rules (DAO 96-40) and the FTAA entered into by then
Government and WMCP grant the Government full control and supervision over all
aspects of planned exploration, development and utilization activities.
Sections 7.8 and 7.9 of the FTAA however are objectionable and void for being
contrary to public policy. Section 7.8 permits the sum spent by government for the
benefit of the contractor to be deductible from the States share in the net mining
revenues since it constitutes unjust enrichment on the part of the contractor at the
52

governments expense. Section 7.9, meanwhile, deprives the Government of its share in
the net mining revenues in the event the foreign stockholders of a foreign mining
company sell 60% or more of their equity to a Filipino citizen or corporation.
Thus, with the exception of Sections 7.8 and 7.9 of the subject FTAA, the FTAA,
RA 7942 and DAO 96-40 are declared constitutional.
The Meaning of Agreements Involving Either Technical or Financial Assistance
A constitutional provision specifically allowing foreign-owned corporation to
render financial or technical assistance in respect of mining or any other commercial
activity was clearly unnecessary; the provision meant to refer to more than mere
financial or technical assistance.
The framers of the Constitution, during its deliberation regarding foreign
investment in and management of an enterprise for large-scale exploration,
development and utilization of minerals spoke about service contracts as the concept
was understood in the 1973 Constitution. It is obvious from their discussions that they
did not intend to ban or eradicate service contracts. Instead, they were intent on crafting
provisions to put in place safeguards that would eliminate the abuses prevalent during
the martial law regime. They were going to permit service contracts with foreign
corporations as contractors but with safety measures to prevent abuses as an
exception to the general norm established in the first paragraph of Section2 of Article
XII, which reserves or limits to Filipino citizens and corporations that are at least 60
percent owned by such citizens the exploration, development and utilization of mineral
or petroleum resources. This was prompted by the perceived insufficiency of Filipino
capital and the felt need for foreign expertise in the EDU of mineral resources.
The drafters, by specifying such agreements involving assistance, necessarily
gave implied assent to everything that these agreements entailed or that could
reasonably be deemed necessary to make them tenable and effective including
management authority with respect to the day-to-day operations of the enterprise, and
measures for the protection of the interests of the foreign corporation, at least to the
extent that they are consistent with Philippine sovereignty over natural resources, the
constitutional requirement of State control, and beneficial ownership of natural
resources remains vested in the State.
It is clear that agreements involving either technical or financial assistance
referred to in paragraph 4 are in fact service contracts, but such new service contracts
are between foreign corporations acting as contractors on the one hand, and on the
other hand government as principal or owner )of the works), whereby the foreign
contractor provides the capital, technology and technical know-how, and managerial
expertise in the creation and operation of the large-scale mining/extractive enterprise,
and government through its agencies (DENR, MGB) actively exercises full control and
supervision over the entire enterprise.
Such service contracts may be entered into only with respect to mineral oils. The
grant of such service is subject to several safeguards, among them: (1) that the service
contract be crafted in accordance with a general law setting standard or uniform terms,
conditions and requirements; (2) the President be the signatory for the government; and
(3) the President report the executed agreement to Congress within thirty days.

53

Ultimate Test: Full State Control


The primacy of the principle of the States sovereign ownership of all mineral
resources, and its full control and supervision over all aspects of exploration,
development and utilization of natural resources must be upheld. But full control and
supervision cannot be taken literally to mean that the State controls and supervises
everything down to the minutest details, and makes all required actions, as this would
render impossible the legitimate exercise by the contractor of a reasonable degree of
management prerogative and authority, indispensable to the proper functioning of the
mining enterprise. Also, the government need not micro-manage the mining operations
and day-to-day affairs of the enterprise in order to be considered as exercising full
control and supervision.
The States full control and supervision over mining operations are ensured
through the different provisions in RA 7942. The government agencies concerned are
empowered to approve or disapprove the various work programs and corresponding
minimum expenditure commitments for each of the exploration, development and
utilization phases of the enterprise. Once they have been approved, the contractors
compliance with its commitments therein will be monitored. The contractor is also
mandated to open its books of accounts and records for scrutiny, to enable the State to
determine if the government share has been fully paid. The State may likewise compel
compliance by the contractor with mandatory requirements on mine safety, health and
environmental protection, and the use of anti-pollution technology and facilities. The
contractor is also obligated to assist the development of the mining community, and pay
royalties to the indigenous peoples concerned. And violation of any of FTAAs terms and
conditions, and/or noncompliance with statutes or regulations, may be penalized by
cancellation of the FTAA. Such sanction is significant to a contractor who may have yet
to recover the tens or hundreds of millions of dollars sunk into a mining project.
Overall, the State definitely has a pivotal say in the operation of the individual
enterprises, and can set directions and objectives, detect deviations and noncompliance by the contractor; and enforce compliance and impose sanctions should the
occasion arise. Hence, RA 7942 and DAO 96-40 vest in government more than a
sufficient degree of control and supervision over the conduct of mining operations.
Section 3(aq) of RA 7942 was objected to as being unconstitutional for allowing a
foreign contractor to apply for and hold an exploration permit. During the exploration
phase, the permit grantee (and prospective contractor) is spending and investing
heavily in exploration activities without yet being able to extract minerals and generate
revenues. The exploration permit issued under Section 3 (aq), 20 and 23 of RA7942,
which allows exploration but not extraction, serves to protect the interests and rights of
the exploration permit grantee (and would-be contractor), foreign or local. Otherwise,
the exploration works already conducted, and expenditures already made, may end up
only benefiting claim-jumpers. Thus, Section 3 (aq) of RA 7942 is not unconstitutional.
The provisions of the WMCP FTAA, far from constituting a surrender of control
and a grant of beneficial ownership of mineral resources to the contractor in question,
vest the State with control and supervision over practically all aspects of the operations
of the FTAA contractor, including the charging of pre-operating and operating expenses,
and the disposition of mineral products.

54

There is likewise no relinquishment of control on account of specific provisions of


the WMCP FTAA.
The FTAA provisions do not reduce or abdicate State control: No Surrender of
Financial Benefits
The second paragraph of Section 81 of RA7942 has been denounced for
allegedly limiting the States share in FTAAs with foreign contractors to just taxes, fees,
and duties, and depriving the State of a share in the after-tax income of the enterprise.
However, the inclusion of the phrase among other things in the second paragraph of
Section81 clearly and unmistakably reveals the legislative intent to have the State
collect more than just usual taxes, duties and fees.
Thus, DAO 99-56, the Guidelines Establishing the Fiscal Regime of Financial or
Technical Assistance Agreements spells out the financial benefits government will
receive from an FTAA, as consisting of not only a basic government share, comprised of
all direct taxes, fees and royalties, as well as other payments made by the contractor
during the term of the FTAA, but also an additional government share, being a share in
the earnings or cash flows of the mining enterprise, so as to achieve a fifty-fifty sharing
of net benefits from mining between the government and the contractor.
The basic government share and the additional government share do not yet
take into account the indirect taxes and other financial contributions of mining projects,
which are real and actual benefits enjoyed by the Filipino people; if these are taken into
account, total government share increases to 60 percent or higher (as much as 77
percent, and 89 percent in one instance) of the net present value of total benefits from
the project.
The third or last paragraph of Section 81 of RA7942 is slammed for deferring the
payment of the government share in FTAAs until after the contractor shall have
recovered its pre-operating expenses, exploration and development expenditures.
Allegedly, the collection of the States share is rendered uncertain, as there is no time
limit in RA 7942 for this grace period or recovery period. But although RA7942 did not
limit the grace period, the concerned agencies (DENR and MGB) in formulating the
1995 and 1996 Implementing Rules and Regulation provided that the period of
recovery, reckoned from the date of commercial operation, shall be for a period not
exceeding five years, or until the date of actual recovery, whichever comes earlier.
Since RA 7942 allegedly does not require government approval for the preoperating, exploration and development expenses of the foreign contractors, it is feared
that such expenses could be bloated to wipe out mining revenues anticipated for 10
years, with the result that the States share is zero for the first 10 years. The argument is
based on incorrect information. Under Section 23 of RA 7942, the applicant for
exploration permit is required to submit a proposed work program for exploration,
containing a yearly budget of proposed expenditures, which the State passes upon and
either approves or rejects; if approved, the same will subsequently be recorded as preoperating expenses that the contractor will have to recoup over the grace period.
The Government is able to know ahead of time the amounts of pre-operating and
other expenses to be recovered, and the approximate period of time needed therefore
because under Section 24, when an exploration permittee files with the MGB a
declaration of mining project feasibility, it must submit a work program for development,
55

with corresponding budget, for approval by the Bureau, before government may grant
an FTAA or MPSA or other mineral agreements. The government has the opportunity to
approve or reject the proposed work program and budgeted expenditures for
development works, which will become the pre-operating and development costs that
will have to be recovered.
Moreover, there is no concrete basis for the view that, in FTAAs with a foreign
contractor, the State must receive at least 60 percent of the after-tax income from the
exploitation of its mineral resources, and that such share is the equivalent of the
constitutional requirement that at least 60 percent of the capital, and hence 60 percent
of the income, of mining companies should remain in Filipino hands. Even if the State is
entitled to a 60 percent share from other mineral agreements (CPA, JVA and MPSA),
that would not create a parallel or analogous situation for FTAAs. .
The Charter did not intend to fix an iron-clad rule of 60 percent share, applicable
to all situations, regardless of circumstances. The terms and conditions of petroleum
FTAAs cannot serve as standards for mineral mining FTAAs, because the technical and
operational requirements, cost structures and investment needs of off-shore petroleum
exploration and drilling companies do not have the remotest resemblance to those of
on-shore mining companies. To avoid compromising the States full control and
supervision over the exploitation of mineral resources, there must be no attempt to
impose a minimum 60 percent rule. It is sufficient that the State has the power and
means, should it so decide, to get a 60 percent share (or greater); and it is not
necessary that the State does so in every case.

56

LEPANTO CONSOLIDATED MINING CO., VS. WMC RESOURCES


507 SCRA 315
Facts:
Respondent WMC Resources International through its local subsidiary Western
Mining Corporation Philippines entered into a contract denominated as Tampakan
Option Agreement and as such it acquired mining claims in Tampakan, South
Cotobato. The agreement underwent several amendments and Tampakan Companies
were then given preferential option to acquire the shares of WMC in WMCP Hillcrest
Inc., in case the former decides to sell them. However WMC sold to Lepanto its shares
and when Tampakan availed of their preferential right to purchase, the same stocks
which were already sold to Lepanto were again sold to Tampakan. Tampakan then
informed the Mines and Geosciences Bureau for the enforcement of the sale and it was
opposed by Lepanto and instead requested for the approval of its acquisition of the
same shares. Lepanto filed an action before the RTC against WMC, WMCP, and
Tampakan. A motion to dismiss was filed by Tampakan on the ground of forum shopping
for there is still a pending action before the MGB.
Issue:
Whether or not the case shall be dismissed due to alleged forum shopping by
Lepanto.
Held:
Yes, for the agreements were already brought before the MGB and the issues
brought before it, which involves the agreements between Lepanto and WMC and WMC
and Tampakan as to the transfer of shares representing ownership of mining rights and
interests over mining agreements, are well within the jurisdiction of MGB. The reliefs
prayed for by Lepanto from both the MGB and the RTC are also identical thus the
elements of forum shopping are present in this case-for it involves same transactions,
essential facts, identical causes of action, subject matter, and issues. Lepanto also
failed to exhaust administrative remedies when it went straight to court without awaiting
the resolution of the MGB.

57

BENGUET CORPORATION VS. DENR


545 SCRA 196
Facts:
Benguet Corp. and J.G. Realty entered into a RAWOP in 1987 wherein it was
acknowledged that J.G. Realty owns four mining claims-Bonito I, II, III, IV totaling to
288.8656 hectares located in Barangay Luklukan, Sitiong Bagong Bayan, Municipality
of San Jose, Camarines Norte. In 1989, Antonio Tanchuling the Executive Vice
President of Benguet Corp. informed J.G. Realty through a letter of its intention to
develop the mining claims. However in 1999, J.G. Realty through a letter sent by its
President Johnny L. Tan informed Benguet Corp. that it was terminating the RAWOP
due to the following: that Benguet Corp. failed to perform their obligations under the
RAWOP such as to undertake works within 2 years from the execution of agreement;
that they violated the contract by allowing high graders to operate on their claim; that
there was no stipulation provided as to the term of the RAWOP; and that there was nonpayment of royalties as agreed by them. Therefore J.G Realty filed a petition for
cancellation of the RAWOP before the Legaspi City Panel of Arbitrators, Region V that
declared the RAWOP cancelled which decision was affirmed by MAB.
Issue:
Whether or not POA had the jurisdiction to decide over the controversy
Held:
No, there should have been a voluntary arbitration first before bringing such
controversy before the POA for Sections 11.01 and 11.02 of the RAWOP provided that
any disputes or disagreements between Benguet and the Owner with reference to the
agreement which cannot be amicably settled shall not be a cause of any action of any
kind in any court or administrative agency but shall instead be referred to a Board of
Arbitrator, and that no court action shall be instituted by the parties unless to have the
decision of the Arbitrators enforced. The Court held that stipulations regarding voluntary
arbitration as valid contractual stipulations and as such the parties must adhere to it. In
the event that a case subject to voluntary arbitration was brought before courts or quasijudicial agencies, the court or quasi-judicial agency must determine whether the
stipulation is sufficient and effective and if found to be so, it shall then order the
enforcement of such provision.
However in this case, estoppel sets in; the Court ruled that Benguet can no
longer question the jurisdiction of POA and MAB, for it should have immediately
challenged the jurisdiction of POA under a special civil action for certiorari when it took
cognizance of the dispute; seven years have already lapsed from the institution of the
action and to redo the proceedings would go against the speedy and efficient
administration of justice

58

METRO ILOILO WATER DISTRICT VS. COURT OF APPEALS


454 SCRA 249
Facts:
Metro Iloilo Water District (MIWD), which was granted water rights to extract and
withdraw ground water within its jurisdiction. MIWD filed an injunction case against
private respondents (Nava et al.) for withdrawing ground water within the formers
jurisdiction without securing a water permit from NWRC, which was still being done
upon filing the petition. Private respondents averred that the RTC had no jurisdiction
over the matter, as the cases were within the original and exclusive jurisdiction of the
National Water Resources Council (Water Council). The RTC dismissed the petitions
saying it had no jurisdiction and that MWID failed to exhaust administrative remedies.
The CA affirmed the RTC Decision, saying that the NWRC has jurisdiction to hear and
decide disputes relating to appropriation, utilization and control of water which was the
subject matter of the case.
Issue:
Whether or not NWRC had jurisdiction over the case.
Ruling:
NO. The petitions filed before the RTC were for the issuance of an injunction
order for private respondents to cease and desist from extracting or withdrawing water
from MIWDs well and from selling the same within its service areas. The petitions focus
on the violations incurred by private respondents by virtue of their alleged unauthorized
extraction and withdrawal of ground water within petitioners service area, visa-avis MIWDs vested rights as a water district. At issue is whether or not private
respondents extraction and sale of ground water within petitioners service area
violated petitioners rights as a water district.
The instant case certainly calls for the application and interpretation of pertinent
laws and jurisprudence in order to determine whether private respondents actions
violate MIWDs rights as a water district and justify an injunction. This issue does not so
much provide occasion to invoke the special knowledge and expertise of the Water
Council as it necessitates judicial intervention. While initially it may appear that there is
a dimension to the petitions which pertains to the sphere of the Water Council, in reality
the matter is at most merely collateral to the main thrust of the petitions.
MIWD had an approved Water Rights Grant from the Department of Public
Works, Transportation and Communications. The trial court was not asked to grant
MIWD the right to use but to compel private respondents to recognize that right. Thus,
the trial courts jurisdiction must be upheld where the issue involved is not the

59

settlement of a water rights dispute, but the enjoyment of a right to water use for which
a permit was already granted.
AMISTOSO VS. ONG AND NERI
130 SCRA 228
Facts:
Amistoso and Neri are owners of adjoining parcels of agricultural land. An
irrigation canal traverses the land of Neri through which irrigation water from the Silmod
River passes and and flows to the land of the Amistoso for the latter's beneficial use.
Amistoso filed a complaint for Recognition of Basement with Preliminary Injunction and
Damages against Neri and Ong(cultivator of Neris land) for refusal, despite repeated
demands, to recognize the rights and title of the former to the beneficial use of the water
passing through the irrigation canal and to have Amistoso's rights and/or claims
annotated on the Certificate of Title of Neri. Neri denied any right of Amistoso over the
use of the canal, nor was there any contract, deed or encumbrance on their property
and assert that they have not performed any act prejudicial to the petitioner that will
warrant the filing of the complaint against them.
Neri asserts that the complaint should be dismissed because Amistosos claim is
based on his right to use water coming from the Silmod River and prays that Amistosos
right to the utilization thereof be respected and not be disturbed and/or obstructed by
Neri. The dispute is thus on the use, conservation and protection of the right to water
and the annotation is merely the relief prayed for on the basis of the claim to the use
and protection of water passing through the land of Neri. And since the controversy
hinges on the right to use and protect the water from the Silmod River that passes on
the land of Neri to Amistoso's property, the proper authority to determine such a
controversy is the National Water Resources Council, which is vested with exclusive
jurisdiction over such question. The trial court dismissed Amistosos complaint for lackof
jurisdiction.
Issues:
1. Whether or not Amistoso has the right over the use of the canal. YES.
2. Whether or not National Water Resources Council has exclusive jurisdiction over
the matter. NO.
Ruling:
Based from the stipulation of facts between the parties, Neri admits that
Amistoso , has an approved Water Rights Grant issued by the Department of Public
Works, Transportation and Communications. Neri contends that the said grant does not
pertain to the beneficial use of irrigation water from Silmod River. The records, however,
do not show any other irrigation water going to petitioner's property passing thru
60

respondents' lot aside from that coming from the Silmod River, making Neris allegations
invalid.
The record clearly discloses an approved Water Rights Grant in favor of
Amistoso. The grant was made three (3) years before the promulgation of P.D. 1067
(Water Code of the Philippines). The water rights grant partakes the nature of a
document known as a water permit recognized under Article 13 of P.D. 1067. the
WATER RIGHTS GRANT of Amistoso does not fall under "claims for a right to use water
existing on or before December 31, 1974" which under P.D. 1067 are required to be
registered with the National Water Resources Council within two (2) years from
promulgation of P.D. 1067, otherwise it is deemed waived and the use thereof deemed
abandoned.
The grant contradicts the erroneous findings of the respondent Judge, and
incontrovertibly entitles petitioner to the beneficial use of water from Silmod River. That
right is now a. vested one and may no longer be litigated as to bring petitioner's case
within the jurisdiction of the National Water Resources Council. To resurrect that issue
will be violative of the rule on res judicata. Amistoso is not asking the court to grant him
the right to use but to compel Neri to recognize that right and have the same annotated
on the latters TCT. The interruption of the free flow of water caused by the refusal to reopen the closed irrigation canal constituted petitioner's cause of action in the court
below, which decidedly do not fall within the domain of the authority of the National
Water Resources Council.

61

LONEY, ET AL. VS. PEOPLE


482 SCRA 194
Facts:
Loney et al., are officers of Marcopper Mining Corporation ("Marcopper"), a
corporation engaged in mining in the province of Marinduque. In one of Marcoppers
operations, the corporation discharged millions of tons of tailings (mine waste) into the
Boac and Makalupnit rivers. The DOJ separately charged Loney et al. for violating
various laws (The Water Code, The Mining act, RPC, and the National Pollution Control
Decree). Loney et al. moved to quash the informations claiming that the informations
were "duplicitous" as the DOJ charged more than one offense for a single act.
The
MTC held that Loney et al. is liable under the Philippine Mining Act but dismissed the
violation of other laws. On petition to the RTC, it affirmed the decision and ordered the
other charges reinstated. The RTC said that there can be no absorption by one offense
of the three other offenses, as [the] acts penalized by these laws are separate and
distinct from each other. The different laws involve cannot absorb one another as the
elements of each crime are different from one another. On petition to the CA, it affirmed
the decision of the RTC.
Issues:
1. Whether or not all the charges filed against petitioners except one should be

quashed for duplicity of charges and only the charge for Reckless Imprudence
Resulting in Damage to Property should stand. NO.
2. Whether or not Branch 94s ruling, as affirmed by the Court of Appeals,
contravenes People v. Relova.
Ruling:
No Duplicity of Charges
There is duplicity (or multiplicity) of charges when a single Information charges
more than one offense. Under Section 3(e), Rule 117 of the 1985 Rules of Criminal
Procedure, duplicity of offenses in a single information is a ground to quash the
Information. The Rules prohibit the filing of such Information to avoid confusing the
accused in preparing his defense. In this case, however, the prosecution charged each
petitioner with four offenses, with each Information charging only one offense. Thus,
Loney et al. erroneously invoke duplicity of charges as a ground to quash the
Informations
The Filing of Several Charges is Proper
62

The filing of the multiple charges against petitioners, although based on the same
incident, is consistent with settled doctrine that where two different laws (or articles of
the same code) define two crimes, prior jeopardy as to one of them is no obstacle to a
prosecution of the other, although both offenses arise from the same facts, if each crime
involves some important act which is not an essential element of the other. A
comparative analysis between the laws shows that each of these laws on which Loney
et al. were charged, there is one essential element not required of the others. Moreover,
the offenses punished by special law are mala prohibita in contrast with those punished
by the Revised Penal Code which are mala in se.The charge for violation of RPC does
not absorb the charges for the other laws because mala in se felonies cannot absorb
mala prohibita crimes.
People v. Relova not in Point
In the case of People v. Relova, the court held that a person charged with theft of
electric power under the RPC after being acquitted of violating a City Ordinance
penalizing the unauthorized installation of electrical wiring violates the right against
double jeopardy because the act giving rise to the charges was punished by an
ordinance and a national statute, thus falling within the proscription against multiple
prosecutions for the same act. However, such is not the case here because Loney et
al.are being prosecuted for an act or incident punished by four national statutes and not
by an ordinance and a national statute. Although Loney et al. cannot be for multiple
prosecuted for the same offense, they can be charged for offenses arising from the
same incident.

63

MMDA VS. CONCERNED RESIDENTS OF MANILA BAY


574 SCRA 661
Facts:
On January 29, 1999, respondents Concerned Residents of Manila Bay filed a
complaint before the Regional Trial Court (RTC) in Imus, Cavite against several
government agencies, among them the petitioners, for the cleanup, rehabilitation, and
protection of the Manila Bay. They alleged that the water quality of the Manila Bay had
fallen way below the allowable standards set by PD 1152 or the Philippine Environment
Code and that the public officials must be solidarily liable to clean up Manila Bay and
restore its water quality to Class B which makes it fit for swimming, diving, and other
forms of contact recreation.
Issue:
Whether or not the public officials, the petitioners in this complaint, may be
compelled by mandamus to clean up and rehabilitate Manila Bay.
Held:
Yes, mandamus may prosper against the public officials. The Court made a
distinction between petitioners obligation to perform their duties and how they carry out
such duties. While the implementation of MMDAs mandated tasks may entail decisionmaking process, the enforcement of the law or the very act of doing what the law exacts
to be done is ministerial in nature and as such it may be compelled by mandamus. The
Court further held that a perusal of the charters of petitioners would give way to the
conclusion that these government agencies are enjoined as a matter of statutory
obligation to perform certain functions in relation to clean up, rehabilitation, protection,
and preservation of Manila Bay thus they are precluded from choosing not to perform
such duties.

64

PUBLIC HEARING COMMITTEE OF THE LLDA VS. SM PRIME HOLDINGS, INC


GR. NO. 170599
SEPTEMBER 22, 2010
Facts:
The petition arose from an inspection conducted by the Pollution Control Division
of the LLDA of the waste water collected from SM City Manila. The results showed that
the samples obtained failed to conform with the standards laid down by law. LLDA then
informed SM Manila of its violation and directing it to perform corrective measures and
to pay a penalty of P1,000/day of discharging pollutive waste water until full cessation of
the discharge of pollutive waste water.
Issue:
Whether or not LLDA has the power to impose fines and collect the same from
SM Prime Holdings.
Held:
Yes, the LLDA has the power to impose fines in the exercise of its regulatory and
quasi-judicial function with regard to pollution cases in the Laguna Lake region. The
Court held that adjudication of pollution cases generally pertains to Pollution
Adjudication Board, except when a special law provides for another forum. In this case
the LLDAs charter provided for such forum. The Court also ruled that although PAB
assumed powers of National Pollution Control Commission in the adjudication of
pollution cases, this does not preclude the LLDA from assuming jurisdiction of pollution
cases within its area of responsibility and to impose fines as penalty.
In sum, for LLDA to effectively perform their functions it need not be required to
resort to some other authority for the proper remedy or penalty. The intent of the law as
gleaned from Section 4(i) of EO 927 is to clothe the LLDA not only with express powers
granted to it but also those that are implied, incidental, and necessary for the proper
implementation of its purposes and functions.

65

THE ALEXANDRIA CONDOMINIUM CORPORATION (TACC) VS. LAGUNA LAKE


DEVELOPMENT AUTHORITY (LLDA)
599 SCRA 452

Facts:
Philippine Realty and Holdings, Inc. (PhilRealty), which constructed and
developed The Alexandra Condominium Complex, transferred to The Alexandra
Condominium Corporation (TACC) such condominium complex by virtue of a Deed of
Conveyance. Subsequently (after 5 years), Laguna Lake Development Authority (LLDA)
advised TACC that its wastewater did not meet government effluent standards, and
informed TACC that it must put up its own Sewage Treatment Plant (STP) for its effluent
discharge to meet government standards. Since constructing an STP would be
expensive (P15M), TACC tried to experiment with other methods of cleaning its
wastewater. However, the wastewater still failed to meet government standards. For this
violation, LLDA imposed a P1000 daily fine on TACC until the wastewater discharge
complies with the government standard.
TACC then entered into an agreement with World Chem Marketing for the
construction of the STP for P7.5M. LLDA issued an Order requiring TACC to pay the
fine (~P1M) representing the penalty from until the STP was constructed. TACC
requested LLDA to condone the imposition of the penalty of P1,000 per day in
recognition of the remedial and corrective measures it undertook to comply with
government standards. TACC further argues that the non-compliance with government
standards was due to the omission and fault of PhilRealty. This was denied by LLDA.
TACC then filed a petition for certiorari before the Court of Appeals (CA) with a
prayer for the issuance of a temporary restraining order. The CA denied TACCs petition.
Issues:
1. Whether or not the TACC complied with the doctrine of exhaustion of
administrative remedies. NO.
2. Whether or not TACC is the one liable to pay the fine. YES.
Ruling:
Non-Exhaustion of Administrative Remedies

66

The doctrine of non-exhaustion of administrative remedies requires that resort be


first made with the administrative authorities in the resolution of a controversy falling
under their jurisdiction before the controversy may be elevated to a court of justice for
review.11 A premature invocation of a courts intervention renders the complaint without
cause of action and dismissible. 12 In this case, TACC has an administrative recourse
before the DENR Secretary which it should have first pursued before filing a petition for
certiorari before the Court of Appeals. This is because under Executive order No. 149
transferred LLDA from the Office of the President to the DENR "for policy and program
coordination and/or administrative supervision. And although under the same order,
DENR only has administrative power over LLDA, a subsequent EO 192 mandates the
DENR to "promulgate rules and regulations for the control of water, air and land
pollution" and to "promulgate ambient and effluent standards for water and air quality
including the allowable levels of other pollutants and radiations."
Powers of the LLDA to Impose Penalty
LLDA, by virtue of its special charter, has the responsibility to protect the
inhabitants of the Laguna Lake region from the deleterious effects of pollutants
emanating from the discharge of wastes from the surrounding areas. Under Section 4-A
of RA 48501, as amended, LLDA is entitled to compensation for damages resulting from
failure to meet established water and effluent quality standards.
It is clear that the responsibility to comply with government standards lies with TACC,
because PhilRealty turned over the project to TACC five years before LLDA advised
TACC that its wastewater did not meet government effluent standards. If, as claimed by
TACC, the non-compliance was due to the omission and fault of PhilRealty, TACCs
recourse is to file an action, if warranted, against PhilRealty in a proper court. TACC
cannot escape its liability to LLDA by shifting the blame to PhilRealty. Hence, the LLDA
did not abuse its discretion in issuing its 4 September 2003 Order.

67

68

You might also like